[HN Gopher] How many real numbers exist? New proof moves closer ...
       ___________________________________________________________________
        
       How many real numbers exist? New proof moves closer to an answer
        
       Author : theafh
       Score  : 454 points
       Date   : 2021-07-15 15:03 UTC (1 days ago)
        
 (HTM) web link (www.quantamagazine.org)
 (TXT) w3m dump (www.quantamagazine.org)
        
       | daxfohl wrote:
       | A related thing that occurred to me the other day: there's some
       | number in [0, 1] that encodes every state of every possible
       | Turing machine (the number of Turing machines is countable, the
       | duration of its run is countable, and the states at each step are
       | countable, so you can diagonalize that and make a real number out
       | of it). I'm pretty sure you can take that further and show that
       | all possible mathematical proofs, including the Godel unsolvable
       | ones (I think...they're just proofs that are infinitely long but
       | countably so), can also be encoded in a single real number. That
       | makes it feel like the reals are pretty big.
        
         | ttctciyf wrote:
         | This sounds a _little_ like Chaitin 's constant Omega (O),
         | which is very roughly a number in [0, 1] which is the
         | probability that a randomly chosen program (for a universal
         | Turing machine, U) will halt.
         | 
         | This has the interesting property that knowing the first n bits
         | of O allows you to determine whether any program (for U) of
         | length up to n will halt, in other words you could solve the
         | halting problem for programs of length up to n.
         | 
         | As Wikipedia puts it:
         | 
         | > Because many outstanding problems in number theory, such as
         | Goldbach's conjecture, are equivalent to solving the halting
         | problem for special programs (which would basically search for
         | counter-examples and halt if one is found), knowing enough bits
         | of Chaitin's constant would also imply knowing the answer to
         | these problems. But as the halting problem is not generally
         | solvable, and therefore calculating any but the first few bits
         | of Chaitin's constant is not possible, this just reduces hard
         | problems to impossible ones, much like trying to build an
         | oracle machine for the halting problem would be.
         | 
         | - https://en.wikipedia.org/wiki/Chaitin%27s_constant
        
         | Rinum wrote:
         | I think you'll like this - https://youtu.be/HeQX2HjkcNo
        
       | bmitc wrote:
       | If people are interested in this stuff, there's a great course
       | called _Paradox and Infinity_ going on right now on edX. You've
       | missed the first two homework assignments, but there's still time
       | to get going in the course.
       | 
       | The course is based on or supported by the book _On the Brink of
       | Paradox_.
        
         | bmitc wrote:
         | Some links:
         | 
         | https://www.edx.org/course/paradox-and-infinity
         | 
         | https://ocw.mit.edu/courses/linguistics-and-philosophy/24-11...
         | 
         | https://www.amazon.com/Brink-Paradox-Highlights-Intersection...
        
       | jaywalk wrote:
       | I'm not trying to be flippant, although it may come off that way:
       | why does any of this matter?
        
         | gorgoiler wrote:
         | We don't know but looking into the past hints at the future.
         | 
         | Cantor, Hilbert and Godel gave us Church who gave us Turing.
         | Turing and Flowers gave us the machines as well as the theory.
         | All of them put together gave us type systems and types are how
         | you formally prove that your 747 software is free of, if not
         | all bugs, then at least certain large classes of error.
         | 
         | There is a clear line of connections from Cantor (1890s) to
         | jumbo jet fly-by-wire (1990s.)
         | 
         | Countability of sets and Cantor's diagonal argument -- the
         | subject of the first part of this article -- are some of the
         | first topics teenagers learn about in high school CS (if you're
         | lucky and on a very modern course) or CS101 (if you're at a
         | University.) Types _are_ sets.
         | 
         | Who knows what today's mathematics will bring us in the year
         | 2121?
        
           | mmastrac wrote:
           | I'd love to see James Burke tackle this line.
        
           | lupire wrote:
           | Eh, this argument kind of falls apart with higher infinities,
           | where the numbers you are studying are bigger than the power
           | set of all interactions among all particles in all possible
           | universes.
        
         | [deleted]
        
         | shockeychap wrote:
         | Why spend time on it? To borrow from George Mallory: Because
         | it's there.
        
         | pjfin123 wrote:
         | I understand the diagnol proof but not a lot of the rest. If
         | you assume only computable numbers exist though a lot of this
         | seems to really not matter.
        
           | lupire wrote:
           | Even if only computable numbers exist, it's inconvenient, to
           | do math on them in bulk, so it helps to have a set of
           | virtual/potential numbers (misleadingly named the "real"
           | numbers, even though almost all of them (100%) will never be
           | realized in any way).
        
         | helen___keller wrote:
         | Others have addressed why it matters (or doesn't matter) when
         | viewed from outside mathematics. But within mathematics,
         | unsolved problems usually matter for two reasons:
         | 
         | (1) When lots of really smart people spend lots of time trying
         | to solve something, and fail to do so, it becomes even more
         | interesting for other smart people. A well-known example is the
         | Collatz Conjecture[0] which, by most accounts is a meaningless
         | problem, but endlessly fascinates many for its renowned
         | difficulty.
         | 
         | (2) In mathematics, you often stumble upon problems that are
         | equivalent to other problems. If X is difficult to solve so you
         | give up and go work on Y, only to find out that Y as a problem
         | is basically the same to X, or they have some other intimate
         | relationship, and it makes X that much more tantalizing. A
         | common example for computer scientists is the whole class of NP
         | complete problems[1] for which we don't know if there exists a
         | polytime algorithm to decide. Every time we discover a new
         | problem that lives in the realm of NP-complete, the P vs NP
         | problem gets a little more tantalizing.
         | 
         | [0] https://en.wikipedia.org/wiki/Collatz_conjecture
         | 
         | [1] https://en.wikipedia.org/wiki/NP-completeness
        
           | sillysaurusx wrote:
           | This is an excellent answer. I have a mostly unrelated
           | question. In the case of the Collatz conjecture, it seems all
           | but proved:
           | 
           | > If the conjecture is false, it can only be because there is
           | some starting number which gives rise to a sequence that does
           | not contain 1. Such a sequence would either enter a repeating
           | cycle that excludes 1, or increase without bound. No such
           | sequence has been found.
           | 
           | There are lots of histograms and empirical data supporting
           | the conjecture.
           | 
           | My question is, why is this empirical data not "good enough"
           | for mathematicians? As something closer to a physicist, I do
           | wonder what the fascination is with trying to prove
           | conclusively that 3n+1 xor n/2 will eventually encounter 1.
           | It seems a bit like trying to prove conclusively that the
           | gravitational constant is so-and-so, when it seems the best
           | you can do is to measure it as precisely as possible:
           | 
           | > Jeffrey Lagarias stated in 2010 that the Collatz conjecture
           | "is an extraordinarily difficult problem, completely out of
           | reach of present day mathematics."
           | 
           | As someone who loved mathematics but was never much good at
           | it, what's the fascination here? (I'm only trying to
           | understand the motivations of people smarter than I am.)
        
             | stonemetal12 wrote:
             | It is simple empirical data isn't proof. It is as if you
             | were testing primes 2 yes, 3 yes, 4 no. Here is empirical
             | data that only 2 primes exist. Should we believe it?
             | 
             | Sure for Collatz we have tested more than 3 numbers but we
             | can't guaranty it is true until we test all infinity of
             | them, or have a proof that doesn't rely on empirical data.
             | To do otherwise would be to look like a fool when someone
             | got around to testing N+1 and it no longer being true.
        
             | Ivoirians wrote:
             | They've verified that the Collatz conjecture holds for all
             | numbers up to ~2^68, but that's precisely 0% of all the
             | numbers that need to be checked.
             | 
             | But more importantly, the goal of (pure) mathematics isn't
             | to declare truths. If you had a machine from God himself
             | that outputted True or False for theorems you put in, that
             | wouldn't demotivate (pure) mathematicians from doing the
             | work they're doing. Understanding the reason things are the
             | way they are (and being able to share those understandings)
             | is the purpose of math. I'd be willing to wager that almost
             | every mathematician would rather have a proof that Collatz
             | holds for all numbers divisible by 17 rather than a
             | definitive yes/no answer to whether it's true or not,
             | because the former would lend much more illumination to the
             | secrets behind the problem, and would lead to new, more
             | interesting mathematical methods and disciplines. The
             | latter would be a fun fact to share at parties.
        
               | schoen wrote:
               | Though note that that True/False machine could also
               | quickly lead to the discovery of actual proofs, for
               | example by making it extremely efficient to search for
               | counterexamples. "The Collatz conjecture is true for all
               | n <= some_huge_number." Also perhaps by making it
               | extremely efficient to search for correct proofs in a
               | lexicographically ordered list of proofs. "The shortest
               | valid proof of the Collatz conjecture in the list of all
               | proofs in my formalism is before proof number
               | some_huge_number."
               | 
               | Although I think the basic version of the machine is
               | "just" the first Turing jump oracle
               | 
               | https://en.wikipedia.org/wiki/Turing_jump
               | 
               | -- it depends on how you formalize the inputs to the
               | machine, right? -- so maybe mathematicians would still be
               | busy afterward. :-) Maybe the machine is an oracle with
               | _infinite_ Turing degree?
        
               | Ivoirians wrote:
               | This is true, a universal oracle would probably lead to
               | some chaos in the world's computer science departments. I
               | wonder how many mathematicians would switch over to help
               | formalize and solve their fields, and how many would
               | stick to their pencils and paper :P
        
               | sillysaurusx wrote:
               | > They've verified that the Collatz conjecture holds for
               | all numbers up to ~2^68, but that's precisely 0% of all
               | the numbers that need to be checked.
               | 
               | This is the crux of what has me looking like a fool to
               | every mathematician in the thread, but I don't mind: why
               | is 2^68 0% of the "numbers that need to be checked"? From
               | a physicist standpoint, you can do a lot with numbers
               | from 0 to 2^68. After all, 64-bit floats are quite
               | useful. Is there 0% value in proving the Collatz
               | conjecture for all possible numbers one might want to use
               | in a normal programming language without big number
               | libraries?
               | 
               | I know the question must sound pretty crude, but it's
               | also a source of mystery. Mathematicians are so obsessed
               | with exactness. Is there no room for empirical analysis
               | in number theory?
               | 
               | In other words, number theory relies on certain
               | assumptions. What if one of your assumptions is "a number
               | system from 0 to 2^64"? Why is there no value in that?
        
               | Ivoirians wrote:
               | >Why is there no value in that?
               | 
               | Because it's uninteresting. The point of pure math is not
               | to be "useful", it's to be interesting. The Collatz
               | Conjecture is (as yet) a completely useless result. Like
               | I said, if God himself came down and told the world "The
               | Collatz Conjecture is true", all we'd get is a useless
               | piece of trivia. "The Collatz Conjecture is true for the
               | first 2^68 natural numbers" is even more worthless than
               | that. Maybe it'd be useful if we had an application for
               | it, but for context, many pure mathematicians are quite
               | derisive at the idea that their work should have
               | practical applications.
               | 
               | Here's a digression, a simple math problem. If you take a
               | checkerboard and remove two opposite corner squares, can
               | you tile the remaining 62 squares with 31 dominoes?
               | 
               | You can probably write a program that can exhaustively
               | churn through all the possible arrangements of dominoes
               | in a checkerboard, and it'll spit out the answer (it's
               | "no"). But is that interesting? No. This is a boring
               | fact, "if you take a checkerboard and remove the two
               | opposite corners you can't tile the remaining squares
               | with 31 dominoes". No one cares about that.
               | 
               | But, here's a proof that this is true. If you look at the
               | colors of each square on a checkerboard, there are 32
               | black squares and 32 white squares. When you remove the
               | two opposite corner squares, you're removing two squares
               | of the same color. So you have 30 black squares and 32
               | white squares left (or the converse). Meanwhile, every
               | domino takes up one black square and one white square. So
               | no matter how you place 31 dominoes, they should cover 31
               | black squares and 31 white squares. Therefore, we've
               | proven the tiling is impossible.
               | 
               | That's somewhat interesting. You have an easily
               | understandable argument for why the fact is true, and you
               | have an application of a method (here, invariants) for
               | looking at other math problems. Plus, it's kinda fun and
               | satisfying and "elegant" to solve a problem like this.
               | The proof is much, much more interesting than knowing the
               | answer to the problem. Hopefully this helps convey that.
        
               | kd0amg wrote:
               | > why is 2^68 0% of the "numbers that need to be
               | checked"?
               | 
               | The vast majority of natural numbers are larger than
               | 2^68. Only 2^68 of them are less than 2^68, but
               | infinitely many of them are greater.
        
             | big_curses wrote:
             | I'll give a perspective, although my specific knowledge of
             | math is not very deep at all, I've thought a lot about
             | concepts and abstractions.
             | 
             | I think the difference here is that math is much more
             | abstract than physics. The concepts backing math are built
             | off of very low level abstractions about the world. For
             | math, over a long period of time more and more
             | relationships and rules were deduced from what had already
             | been induced from reality. And if you CAN deduce, you
             | should likely, as it provides a proof for some idea or
             | concept that induction never could (but only if your
             | previous inductions and deductions were accurate).
             | 
             | Physics on the other hand, cannot deduce as readily. It is
             | primarily based in the realm of gathering more and more
             | data from the world and observing physical relationships
             | firsthand. This cannot be done in abstract math because
             | abstract math does not exist in reality. There is nothing
             | to observe, it is mostly abstractions based on lower level
             | observations in reality. For example, you can measure the
             | effects of gravity firsthand, but you cannot measure
             | infinity, a mathematical abstraction. Infinity does not
             | exist in reality. It is simply a useful abstraction for
             | things that are too large or small for us to meaningfully
             | measure.
        
             | Sharlin wrote:
             | > There are lots of histograms and empirical data
             | supporting the conjecture.
             | 
             | There's a very interesting implicit question there: _why
             | should counterexamples be small?_ [1][2] Certainly,
             | counterexamples to many conjectures about infinite sets can
             | be found with a brute-force search, even if the problem is
             | merely semidecidable. But isn 't that simply an example of
             | selection bias?
             | 
             | There is a certain subjective beauty in having
             | counterexamples like 9 or 341 or even 23338590792. But is
             | that just anthropocentrism? After all, no matter how many
             | cases we check, we have made absolutely no progress in
             | exhausting the whole set of natural numbers! We can never
             | reach even reasonably easily constructable numbers like
             | 3|||3 (using Knuth arrow notation [3]), and still almost
             | all[4] natural numbers are bigger than that.
             | 
             | In physics, there's an (often implicitly made) assumption
             | that more evidence in support of a hypothesis makes it more
             | likely that the hypothesis is supported by any future
             | evidence as well. But why should we be able to make that
             | assumption? We do, because it seems to work, but why should
             | it still work tomorrow? This is, of course, the famous
             | philosophical problem of induction [5]. But math is
             | basically what happens when you _explicitly reject
             | inductive reasoning_ and then start to explore the space of
             | things that can still be reached, using purely deductive
             | reasoning!
             | 
             | [1] https://math.stackexchange.com/questions/449886/the-
             | largest-...
             | 
             | [2]
             | https://math.stackexchange.com/questions/111440/examples-
             | of-...
             | 
             | [3] https://en.wikipedia.org/wiki/Knuth%27s_up-
             | arrow_notation
             | 
             | [4] https://en.wikipedia.org/wiki/Almost_all
             | 
             | [5] https://en.wikipedia.org/wiki/Problem_of_induction
        
             | ivanbakel wrote:
             | >My question is, why is this empirical data not "good
             | enough" for mathematicians?
             | 
             | There are two reasons: firstly, because mathematics is not
             | an empirical discipline (well, unless you're a number
             | theorist...), so it is possible to be certain of
             | mathematical truth, unlike the inherent uncertainty of
             | physical truth; secondly, because every finite bound on the
             | natural numbers may as well be 0 when compared to the
             | numbers that remain.
             | 
             | There is simply no way to take any empirical measurements
             | of the natural numbers (or the reals) that would let you
             | estimate anything "for all numbers", since the proportion
             | of numbers you failed to sample is infinite.
             | 
             | You may be interested in reading the answers to this
             | question:
             | https://math.stackexchange.com/questions/514/conjectures-
             | tha... which describes some problems which seemed true "up
             | to some large number", but later turned out to be false.
        
               | sillysaurusx wrote:
               | https://math.stackexchange.com/questions/514/conjectures-
               | tha... was excellent. Thank you.
               | 
               | > n^17+9 and (n+1)^17+9 are relatively prime
               | 
               | > The first counterexample is
               | n=8424432925592889329288197322308900672459420460792433
               | 
               | I think this helped me appreciate the difficulty of being
               | satisfied with the Collatz conjecture.
        
             | helen___keller wrote:
             | Well, firstly there are "important numbers" much bigger
             | than the range we've empirically tested, so our empirical
             | results aren't actually good enough. If there was some deep
             | relationship with number theory, maybe it just happens that
             | a number in this range is the first to break the pattern,
             | which could be a deep and beautiful result.
             | 
             | But also, it's just the nature of mathematics, proving what
             | is true is just as important (if not more so) than knowing
             | what is true.
             | 
             | From a practical perspective, doing mathematics, you often
             | don't _really_ grok why something is true until you prove
             | it.
             | 
             | From a philosophical perspective, there's not much in the
             | universe we can know for sure (as you mention with the
             | gravitational constant), but a mathematical proof we know
             | for sure. That's the beauty of it.
             | 
             | if X is true and the implication (X => Y) is true, then it
             | must be that Y is true. By definition of a logical
             | implication.
             | 
             | Add in a whole bunch of axioms and suddenly you can take
             | these simple logical atoms and build them into a field that
             | spans the working tools of every engineer and beautiful
             | arcane subjects like set theory. And every single thing we
             | prove, we know to be true, as we build it from logical
             | atoms that can be traced back to definitions and axioms.
             | 
             | It "could be" that for certain types of matter or at
             | certain distances, the gravitational constant changes. It
             | cannot be that there exists a bijection between the natural
             | numbers and the real numbers under ZFC. Cantor's
             | diagonalization argument *proves* it.
        
             | myWindoonn wrote:
             | Some theoretical physicist (Aaronson?) once quipped
             | something like, "If physicists had discovered P vs. NP,
             | then they'd have said P != NP and given a Nobel for it. And
             | then if it turned out P = NP, then they'd give another
             | Nobel."
             | 
             | Another, more important reason is that we know that the
             | Collatz Conjecture is a single slice of a Turing-complete
             | question about dynamical systems. Trying to find a complete
             | proof expands our knowledge about the bridge between
             | dynamical systems and the natural numbers. Such
             | explorations were essential to founding modern physics in
             | terms of dynamics and conservation laws.
        
             | hvocode wrote:
             | I think others addressed the "why proof not empirical data"
             | question well, but one additional point. A logical proof is
             | only as solid as its weakest component. In mathematics any
             | result that has been proven may be used in a component of
             | another proof. If you have a result that is based on
             | empirical evidence, then every result proven based on that
             | also inherits that empirical evidence as part of its
             | foundation. The reason we don't do this is that if, by some
             | weird chance, the original result that used empirical data
             | instead of proof is shown to be false, then every result
             | built upon it is invalidated and needs to be revisited.
             | That would be a mess. Sticking to requiring formal proofs
             | at least reduces that possibility - although it is still
             | entirely possible for a proof to have a subtle mistake as
             | well, which would have a similar cascading effect on
             | results built upon it.
        
             | svat wrote:
             | No amount of empirical data is ever "good enough" for the
             | mathematical standard of proof. (It may be enough for
             | mathematicians to "believe" something in some informal
             | sense, but not enough to consider it "proved".) You can see
             | some examples at the answers to these questions; maybe at
             | least one of them will be interesting to you:
             | 
             | - https://math.stackexchange.com/questions/514/conjectures-
             | tha...
             | 
             | - https://math.stackexchange.com/questions/111440/examples-
             | of-... (and maybe
             | https://mathoverflow.net/questions/11517/computer-algebra-
             | er... )
             | 
             | - https://mathoverflow.net/questions/15444/examples-of-
             | eventua...
             | 
             | (There are over a hundred examples at those questions; I
             | guess this counts as a lot of empirical data that empirical
             | data is not enough! However, sometimes empirical data _can_
             | be enough for a mathematical proof; for instance if you
             | know somehow that a polynomial f is of degree less than n
             | and you have shown that f(x)=g(x) at n distinct points, you
             | can indeed conclude that f=g and so on -- see the  "Proofs
             | by Example?" section of the first "Proof Machines" chapter
             | of the book "A=B" available online
             | https://www2.math.upenn.edu/~wilf/AeqB.html .)
        
               | sillysaurusx wrote:
               | That was another question in the back of my mind --
               | famously, the four color theorem was proved by computers
               | through exhaustive analysis (checking every possibility).
               | At the time, it was controversial as a "proof" since it
               | didn't really take the usual form of a proof.
               | 
               | I've often wondered "Why can't we do something like that,
               | but for all instances of things like the Collatz
               | conjecture?" Of course, it's computationally infeasible.
               | But that raises the question: Suppose the four-color
               | theorem was only able to prove 95% of cases rather than
               | 100%. Isn't it at least sort of valuable to do so? Or is
               | that last 5% all the difference?
               | 
               | I suppose what bugs me about math is, physicists are
               | proved wrong all the time. Mathematicians are rarely
               | proved mistaken, because they construct assumptions that
               | you can't disagree with. There's no chance for empirical
               | data to falsify one's assertions.
               | 
               | But that's an odd kind of debate, and not too productive.
               | I just can't help but wonder about stuff like this.
        
               | bordercases wrote:
               | You're adhering to certain misconceptions for no good
               | reason. An exhaustive check of all the cases in the model
               | that supports the four-color conjecture _is_ the
               | "empirical" (here, meaning experimental) data required to
               | falsify conjectures. And even physicists don't deny the
               | value of falsification in narrowing down models to truth.
               | But I think they are much more loose in what they are
               | willing to stipulate as true if it means advancing other
               | conclusions.
               | 
               | Furthermore mathematicians do accept certain kinds of
               | probabilistic arguments, but they need to take a form of
               | proving that an object will be guaranteed to have a
               | certain property as a parameter on the object approaches
               | a statistical limit. So partial evidence can sometimes
               | point to the possible truth of a mathematical conjecture,
               | but you are still burdened with the requirement of
               | deductive demonstration if you desire logical certainty.
               | 
               | The tautological notion you take of math is a half-truth.
               | On the one hand, models in math seek to be sound over the
               | objects they specify. This is opposed to physics where
               | deliberate simplifications must be made to make your
               | idealizations of a system manageable or relevant. At the
               | same time, no mathematical system is complete. So there
               | will be truths that require other axiomatizations to
               | access via proof. This is one source of mathematical
               | creativity requiring judgment beyond following
               | tautologies.
        
               | Sharlin wrote:
               | Math is simply not a science, not in the sense of
               | "natural science". Math research does not follow the
               | scientific method, it is not an iterative effort to
               | construct (wrong, but useful) models of some external
               | universe like science does. Math is a thing of its own,
               | and it's best just to not compare it to physics or
               | biology or gender studies or whatever.
        
               | acchow wrote:
               | "famously, the four color theorem was proved by computers
               | through exhaustive analysis (checking every possibility)"
               | 
               | This is impossible. The plane can be arbitrarily large
               | with an arbitrary number of regions. It can be
               | exhaustively checked for n number of regions up to a
               | certain n. But not for arbitrary n.
        
               | HelloNurse wrote:
               | Not every possible graph, just every "interesting" class
               | of subgraph that can be extended or combined with others
               | to obtain a coloring of any planar graph. These are a
               | large but finite number; if a graph is planar it cannot
               | have too many edges.
        
               | schoen wrote:
               | The four-color theorem had the preliminary challenge of
               | creating a method to identify all the relevant cases.
               | That also required a mathematical theory. (I don't
               | actually understand how it was done!)
               | 
               | If you didn't have that, you would have an infinite
               | search for possible maps that violate the conjecture,
               | because you wouldn't be able to divide them into a finite
               | number of equivalence classes, or enumerate a finite
               | number of possible counterexamples, or whatever.
               | 
               | For Collatz, I don't think we have any lemma that gives a
               | path to saying "a counterexample must be one of these
               | 2100 integers" or the like. So without such a thing,
               | checking every possibility would mean checking _every
               | integer_. (Well, there are definitely lemmas that make it
               | unnecessary to check large numbers of integers, so I
               | should say instead that checking every possibility would
               | still mean checking an infinite number of cases.)
        
               | kevinventullo wrote:
               | As an interesting example where we did have a finite
               | bound that was still out of reach to computationally
               | exhaust, see the Weak Goldbach Conjecture:
               | 
               | https://en.m.wikipedia.org/wiki/Goldbach%27s_weak_conject
               | ure
               | 
               | It was known to have at most finitely many counter-
               | examples by the 1930's, and by the 1950's it was known
               | that the largest counter-example had to be < 3^3^15. In
               | 2002 that was lowered to about 10^1346. Still well out of
               | reach of any computer!
               | 
               | Only in 2012 was an unconditional proof given.
        
               | helen___keller wrote:
               | It can be valuable, but it's not as valuable as covering
               | all cases.
               | 
               | If I could prove collatz for 99% of numbers it would be
               | great.
               | 
               | If i could additionally prove which 99% converge, let's
               | say all numbers not divisible by 100, it would be
               | enormous because even showing this much is likely to be a
               | stepping stone to a full proof (what property is it about
               | the number 100 that excludes all non-divisors from
               | diverging?)
               | 
               | > I suppose what bugs me about math is, physicists are
               | proved wrong all the time. Mathematicians are rarely
               | proved mistaken, because they construct assumptions that
               | you can't disagree with. There's no chance for empirical
               | data to falsify one's assertions.
               | 
               | Right, and that's the whole point and beauty of it.
        
           | thehappypm wrote:
           | I mean, at its simplest, the Collatz conjecture is a type of
           | random walk, with the sequence either exponentially growing
           | or exponentially shrinking, basically at random. It's
           | statistically impossible that that any random walk will
           | continue one direction forever, and once it randomly decays
           | to 1 you're done.
        
             | quickthrower2 wrote:
             | Loops are the other option you'd need to prove don't exist.
             | 
             | I think I can see why trying to solve this problem is
             | popular. It almost feels like it should be easy to solve
             | but it just quite isn't!
        
         | sillysaurusx wrote:
         | It took many years to understand that!
         | 
         | It's not a flippant question at all.
         | 
         | The answer is, it doesn't matter. And that's the joy of it.
         | 
         | It wasn't until I got into ML that I learned the value of doing
         | unimportant work. When you're free to think about
         | inconsequential matters very seriously, you end up discovering
         | so many useful things. It was how I independently rediscovered
         | what is apparently called the Discrete Hartley Transform.
         | 
         | You have a vague notion that what you're doing might be
         | important, but it's not the focus. It matters because it's fun
         | in a way that nothing else can be.
         | 
         | Of course, the more serious folk won't admit it's fun. It's
         | serious work. And they're not wrong; nobody's fooling around.
         | Our time is every bit as precious as an executive's.
         | 
         | But their goal is to change the world. Ours -- or mine, at
         | least -- is to know the truth of a thing that most people don't
         | know.
        
           | mr_gibbins wrote:
           | I agree entirely. Industrially, I work a role within an IT
           | team. My work is, I hope, useful and strives towards a
           | purpose - my employer's purpose/business model.
           | 
           | Do I 'go home' - leave my home office - and think about it?
           | No. Do I muse on how to solve a work-related issue when I'm
           | showering? Also no. Will I forget almost everything about
           | this job as or when I move onto the next? Definitely yes.
           | 
           | But academically, I'm interested, and actively publish albeit
           | modest, low-impact research, in relational databases. I love
           | the set-based approach to all matters SQL and after many
           | years in the field still fool around trying to embellish,
           | attack, improve and invent the core ideas.
           | 
           | Why bother? Even if I come up with some magical new
           | improvement to RDBMSs, it doesn't matter. If I fork MySQL and
           | try out my ideas, no one cares. I'll never get the traction
           | or the FOSS community support, I'll spend more time playing
           | politics and managing collaborators than I'd like (zero) and
           | I enjoy the independent thought that my interests bring.
           | 
           | So my academic work is pointless. But it's meaningful to me.
           | And I think that's what matters. I don't understand one-tenth
           | of the ideas in this article, but that doesn't matter. What
           | matters is that it's interesting to _somebody_.
        
           | fighterpilot wrote:
           | > it doesn't matter
           | 
           | It doesn't matter _yet_. Applications of pure math happen
           | downstream decades or centuries later. It 's hard to predict
           | the impact.
        
             | l33t2328 wrote:
             | I don't like this framing because it implies any piece of
             | pure math becoming useful is just a matter of time, and I
             | see no reason to suspect this is the case.
        
               | hawkice wrote:
               | I think it's more about being open minded to strange
               | uses. Maybe 3% of strange pure math ends up being used
               | for something bigger? But since nobody knows what it'll
               | be, it's good to be curious about all sorts of nonsense.
               | Riemannian geometry was OBVIOUSLY absurd useless pure
               | math sophistry, but then it turned out we lived in it. I
               | think a lot of the work on modular forms and elliptic
               | curves has come across that way too, despite the crypto
               | intersection. The general idea is that, even if it's been
               | a century or two, it's too soon to tell if something is
               | useless, that's solid. And formalization of infinity has
               | been useful in some practical exploration of proof
               | systems used by humans. It might be the future of
               | programming, who knows?
        
               | bmitc wrote:
               | > I see no reason to suspect this is the case
               | 
               | Isn't history enough reason?
        
               | jonnycomputer wrote:
               | I've read lots of math papers that were a complete waste
               | of time because they were banal. The only "innovation"
               | was to use different words to describe the same thing.
               | New mathematics is actually pretty damn hard I suspect.
        
               | erichahn wrote:
               | No history actually tells you that only 1% of produced
               | maths is useful. You only see the good stuff not the ugly
               | stuff.
        
               | bmitc wrote:
               | I don't know why people are throwing around percentages
               | so much in these replies. Percentage of what? Plus, the
               | body of work of mathematics continues to grow all the
               | time.
               | 
               | The core of the original comment that started this chain
               | was:
               | 
               | > Applications of pure math happen downstream decades or
               | centuries later.
               | 
               | My argument was that history shows that a surprising
               | amount of mathematics does eventually trickle down into
               | applications. I don't think anyone is arguing all of
               | mathematics eventually sees an application.
               | 
               | Again, my point was that history does indeed seem to show
               | that "applications of pure math happen downstream decades
               | or centuries later". As the original commenter said, it's
               | hard to predict what will be the next thing to be
               | applied.
        
               | l33t2328 wrote:
               | In addition to what the other commenter said, it's
               | important to understand the scale. Cutting edge research
               | 600 years ago was solving cubic equations. Today, cutting
               | edge research requires potentially years of advanced
               | study to understand the objects involved.
               | 
               | There are more mathematicians alive today than at any
               | point in human history, and their work has become
               | specialized to a tremendous degree. Gone are the days
               | where one mathematician could do substantial new work in
               | a dozen diverse areas.
               | 
               | So, in short, the fact that historically lots of pure
               | math has not found use, the specialized nature of modern
               | research math, the volume of work being out out, and an
               | irreverence for applications(again, this is _pure_ math)
               | leads me to be doubtful that even a moderate amount of
               | modern pure math research will ever be useful in any
               | practical sense.
        
             | dfdz wrote:
             | To follow up on this comment, I think that math should be
             | viewed as a giant tree structure or pyramid
             | 
             | At the bottom are all sorts of applications of linear
             | algebra and optimization (calculus) are related to machine
             | learning, deep learning (AI), physics, engineering etc.
             | 
             | One level up there are all sorts of ideas in algebra and
             | mathematical analysis that are leading to new ideas in
             | linear algebra and optimization (calculus)
             | 
             | Going one level higher, there are ideas in mathematical
             | logic and set theory that are improving our understanding
             | our algebra and analysis.
             | 
             | If you look at a specific result at the very top of the
             | pyramid as ask why does it matter? It is difficult to give
             | a good answer. Clearly we want to base of the pyramid, but
             | do we need to keep building it up? Should we stop at a
             | certain level?
        
           | just_one_time_ wrote:
           | Lololol. U gay.
        
         | Hamlet42 wrote:
         | Some math problems seem uselees when first encountered until
         | they become useful some dacades later. An example of this is
         | knot theory and medicine[1]
         | 
         | [1] https://science.sciencemag.org/content/255/5043/403
        
           | db48x wrote:
           | Also certain of the more recondite aspects of number theory
           | that deal with prime numbers: completely useless right up
           | until RSA was invented; now we use them to protect every
           | webpage we view, every email we send, and every financial
           | transaction we make.
        
         | Y_Y wrote:
         | I think you'd have to say exactly what you mean by "matter".
         | 
         | At one point in my life I felt like the continuum hypothesis
         | was the most important question in the world. Now I'm mostly
         | concerned about finite material issues, e.g. my health. I think
         | a good argument can be made why either of them matter much more
         | than the other.
        
         | soperj wrote:
         | This is kind of the equivalent of asking why does your life
         | matter?
        
         | l33t2328 wrote:
         | It's interesting, and fun.
         | 
         | Most modern pure math is done completely independent of and
         | without regard to applications(if it were, it would be 'applied
         | math').
        
         | bmitc wrote:
         | We know that the cardinality of the natural numbers is less
         | than the cardinality of the real numbers. The Continuum
         | Hypothesis, which is a long unsolved problem, states that there
         | are no sets with cardinality between the two. The posted
         | article states that this new result strengthens the case
         | against the hypothesis, that is that's it's probably false.
         | 
         | All of this is nuanced but is important to mathematics and
         | philosophy.
        
           | thom wrote:
           | Why does "the set containing the natural numbers and a
           | sandwich" not have cardinality between the two?
        
             | knodi123 wrote:
             | Because cardinality of finite sets is intuitive, but
             | cardinality of infinite sets is less intuitive and totally
             | different. The natural numbers plus a sandwich is mixing
             | the two together in a single argument, which doesn't really
             | track.
        
             | bmitc wrote:
             | The non-sandwich analogy is called Hilbert's hotel.
             | 
             | Saying that two sets have the same cardinality is
             | equivalent to them having a bijection between them.
             | 
             | So the claim is that the natural numbers and the natural
             | numbers plus a sandwich have the same cardinality. This can
             | be proved by the bijection:                   0 -> sandwich
             | 1 -> 0         2 -> 1         3 -> 2           .
             | .           .         n -> n-1           .           .
             | .
             | 
             | There is actually more though! If you had an infinite but
             | countable amount of sandwiches (that is a sandwich for
             | every natural number), that plus the natural numbers still
             | has the same cardinality as just the natural numbers.
             | There, the bijection is                   0 -> sandwich_0
             | 1 -> 0         2 -> sandwich_1         3 -> 1         4 ->
             | sandwich_2         5 -> 2           .           .
             | .
             | 
             | No natural number or sandwich is left out by this mapping.
        
               | thom wrote:
               | Because everyone gave me such good and well meaning
               | answers perhaps you'll permit me a follow up.
               | 
               | As I understand it we can say the cardinality of the
               | reals is 2^aleph_0. Why is it cheating to create a
               | bijection thusly:                   0 -> 0         1 ->
               | 1/(2^aleph_0)         2 -> 2/(2^aleph_0)
               | 
               | etc?
        
               | dabitude wrote:
               | Are real numbers uniformly spaced? Or are there clusters?
        
               | bmitc wrote:
               | I am not entirely sure what you wrote or are asking, but
               | if you're interested in this stuff, come take the course
               | (or read the book) I mentioned here:
               | 
               | https://news.ycombinator.com/item?id=27846666
        
               | scapp wrote:
               | Problem 1: 1/(2^aleph_0) isn't a real number. The real
               | numbers don't contain infinitesimals. It's possible to
               | formalize a number that behaves like 1/(2^aleph_0) "ought
               | to" (surreals would be one possible approach), but the
               | result won't be a real number.
               | 
               | Problem 2: There's no natural number that maps to (say)
               | 1. Even if you do allow 1/(2^aleph_0), there's no finite
               | number n that would make n/(2^aleph_0) = 1. With any
               | reasonable definitions of the operations involved here,
               | n/(2^aleph_0) would always be infinitesimal, so it would
               | never equal a non-infinitesimal.
               | 
               | Problem 3: You're still skipping over infinitely many
               | numbers. If 1/(2^aleph_0) is a number (and again, this
               | requires going beyond the real numbers) and 1.5 is a
               | number, then 1.5 * 1/(2^aleph_0) = 1.5/(2^aleph_0) is
               | also a number, but no natural number gets mapped to that.
        
               | drdeca wrote:
               | 1/(2^{\aleph_0}) isn't something that has a clear
               | meaning.
               | 
               | 2^{\aleph_0} is a cardinal number, which isn't really a
               | number in the sense of "an element of a field" or
               | something like that. Dividing by it isn't a well defined
               | thing.
               | 
               | And, you certainly can't just multiply any real number
               | (or, any real number between 0 and 1) by 2^{\aleph_0} and
               | get a different integer as a result.
               | 
               | (Now, if you work in the surreal numbers, you can define
               | things like n/(2^{\aleph_0}) (identifying cardinals with
               | the first ordinal of that cardinality), but these would
               | not be real numbers. They would all be infinitesimal ,
               | smaller than 1/k for all positive integers k, and yet
               | bigger than 0. Similarly in the surreal numbers, you
               | could multiply real numbers between 0 and 1 by
               | 2^{\aleph_0}, but you would get surreal numbers which are
               | larger than every integer (in fact, larger than any
               | countable ordinal))
               | 
               | Summary: What you wrote doesn't define a mapping from the
               | integers to the real numbers . (It can be interpreted as
               | defining a map from integers to something else though.)
        
               | lupire wrote:
               | Don't stop there! Let's have a countably infinite variety
               | of sandwiches, with a countably infinite number of
               | sandwiches of each variety.
               | 
               | Still enough natural numbers to eat them all, one per.
               | 
               | But still not enough sandwiches to feed all the (so-
               | called) real numbers one sandwich each!
               | 
               | But if sandwiches grew on trees, and we had an infinite
               | branching tree with two branches at each branching point,
               | and every branch has a (pair of) sub-branches, then
               | natural numbers could not eat all the sandwiches, and the
               | sandwiches could feel all the (so-called) real numbers.
        
             | fighterpilot wrote:
             | For the same reason that integers have the same cardinality
             | as only odd numbers. You can create a 1:1 mapping between
             | them.
        
               | bmitc wrote:
               | Not just one-to-one (injective) but also onto
               | (surjective). :)
        
             | orbital223 wrote:
             | The cardinality of infinite sets is unintuitive. We can
             | create a one to one correspondence between the
             | Naturals+Sandwich set and the Naturals set (for example,
             | assign sandwich to 0, 0 to 1, 1 to 2, 2 to 3, etc). That
             | means they have the same cardinality.
             | 
             | It's even possible to add an infinite number of elements to
             | an infinite set and retain the same cardinality (the
             | Integers set has the same cardinality as the Naturals set).
        
             | hawkice wrote:
             | Because it's the same size. I can show it's the same size
             | because I can just change the labels on the sets (label
             | change of elements doesn't change group size) and go back
             | and forth. In this case, 0 is the sandwich, and all the
             | nonnegative numbers get relabelled down by one. Going back,
             | sandwiches become 0 and you add one to nonnegative numbers.
             | So they've got to be the same size, if I can go back and
             | forth and everything just gets a new label and I don't miss
             | anything.
        
             | helen___keller wrote:
             | You can create a bijection between that set and the set of
             | natural numbers, defined by the function f:
             | 
             | 0 <-> sandwich
             | 
             | 1 <-> 0
             | 
             | ...
             | 
             | n+1 <-> n
             | 
             | By definition, if you can biject two sets, they have the
             | same cardinality.
        
             | magicalist wrote:
             | One way to say two sets of things are of equal size is if
             | we can pair up all the elements of one set with all the
             | elements of the other.
             | 
             | If we're ok with extending that to sets of infinite things
             | (we can still pair elements of each set, we'd just never be
             | able to finish listing all the pairs), then we can say that
             | the natural numbers and "the set containing the natural
             | numbers and a sandwich" are of equal size because we could
             | pair 1 from the first set with the sandwich from the second
             | set, pair 2 from the first set with 1 from the second set,
             | 3 from the first set with 2 from the second set, etc etc.
             | 
             | There's no element of either set without a match in the
             | other set, so they have the same cardinality as the natural
             | numbers, with or without the sandwich.
        
         | pantulis wrote:
         | Because this advances math. Same could be said for lambda-
         | calculus and yet here we are with FP.
        
         | ssivark wrote:
         | Let me venture a possible application, since there are already
         | many responses celebrating pure mathematics.
         | 
         | As far as we understand, the natural numbers are not sufficient
         | for modeling physical phenomena. The reals/complex while
         | immensely useful also occasionally turn out to be "too
         | complicated" to give theoretical guarantees/proofs of models
         | working well. On the practical side, that means that these
         | models/algorithms can't be guaranteed to not give junk results,
         | while working on the domain of real numbers.
         | 
         | Now, purely speculatively, since the naturals/rationals are
         | aleph0 in size and the reals are aleph2, that means there
         | likely exists a set of numbers of size aleph1 sitting in
         | between the two. What if we could use that set of numbers to
         | construct our physical models? Could we somehow guarantee
         | better behavior... Eg: in quantum mechanics, or field theory,
         | or chaos, etc?! :-)
        
           | drdeca wrote:
           | Whether the cardinality of the reals is aleph 1, aleph 2, or
           | something larger, is independent of ZFC.
           | 
           | In a model of ZFC in which the set of reals has cardinality
           | greater than aleph_1 , I wouldn't be surprised if there is a
           | subfield of the reals of cardinality aleph_1 , but I would be
           | surprised if such a field was useful for things like that.
           | Such a field would, of course, not be complete with respect
           | to the usual metric on the rationals, so we wouldn't have the
           | desired convergence properties. We wouldn't really be able to
           | do infinite sums in it? (Well, perhaps some other sense of
           | infinite summation could be done, but it wouldn't be the
           | usual sense.)
           | 
           | In addition, because such a subfield would only exist as an
           | uncountable proper subfield in some models of ZFC, I find it
           | hard to imagine that it would allow computations that
           | wouldn't otherwise work? I suppose it could motivate some
           | computations which would then also work regardless of what
           | model of ZFC is being used / is true ?
        
       | morpheos137 wrote:
       | It depends on what you define as a number. For example reals and
       | complex have different properties from integers. Is there a
       | mathematical reason why both are considered numbers? You can
       | count (with) integers but not with reals. Thus one appears to be
       | a number while the other appears to be a measure.
        
         | Rerarom wrote:
         | "Number" is not a mathematical term.
         | 
         | "Natural number", "real number", "complex number" are.
         | 
         | The fact that they all contain the string "number" is
         | completely irrelevant.
        
         | kmeisthax wrote:
         | The essential property of anything mathematicians tack the word
         | "number" onto _isn 't_ the fact that there's always a "next"
         | number to count with. And it's very useful to do math with
         | reals, otherwise we wouldn't be able to talk about things like
         | the circumference of a circle, because you need an irrational,
         | transcendental (not the root of a finite rational polynomial)
         | constant.
         | 
         | Generally speaking, a good chunk of the number systems
         | mathematicians worked with involve taking some existing number
         | system (tracing back to the natural numbers), finding some
         | operation that takes you outside that system, and then building
         | a new system to cover that hole. A system that has no such
         | holes given a particular operation is said to "close over" that
         | operation. So, integers close over subtraction; rationals close
         | over division, and complex numbers close over exponentiation.
         | Since all of these systems ultimately derived from operations
         | over the natural numbers, it makes sense to also call them
         | numbers, even though they might lack certain symmetries or
         | properties of simpler number systems.
         | 
         | Also, the word "measure" is already taken by a different
         | concept.
        
         | croes wrote:
         | You are confusing numbers with natural numbers.
         | 
         | "A number is a mathematical object used to count, measure, and
         | label."
        
           | morpheos137 wrote:
           | You can't count reals or complex numbers. Real numbers by
           | definition describe objects with infinite precision. In the
           | real world infinite precision cannot exist therefore real
           | numbers are the limit of an arbitrary precision measuring
           | process not real themselves.
           | 
           | Meanwhile natural numbers like "two" most certainly do exist
           | as a quantitative attribute of a set.
           | 
           | Show me how you can count with reals.
        
             | croes wrote:
             | I don't need to, I can measure and label with real numbers.
             | That's enough for being a number.
        
               | gowld wrote:
               | You'll only ever use 0% of the real numbers for that.
               | Unless you say that you count with "the reals", You don't
               | need and won't use "the reals" for measurement and
               | labelling, 100% of which are indescribable.
        
               | morpheos137 wrote:
               | technically you will use an infintesimally small real
               | number approaching 0%...
        
             | TuringTest wrote:
             | At the end of the day, a number is a _reification_ (or
             | thing-a-fication, watching a process as a separate entity)
             | of the processes of _mapping_ and _ordering_. You may begin
             | with a very simple use of those processes, which get you
             | the natural numbers. But you may use them in more creative
             | ways, which will get you other different classes of
             | numbers. Mathematician love to explore all the implications
             | of using basic processes and combining already defined
             | numbers to create new kinds, never seen before.
             | 
             | For example, if you start with number '1' and apply
             | operator _successor_ (or "adding one more"), you get the
             | *natural numbers*, which are a mapping from the size of
             | sets to strings of operators {'1', 'successor(1)',
             | 'successor(successor(1))', ... }. You can use this process
             | to define an order: number A is smaller than B if you can
             | repeatedly apply _successor_ to A and generate B in a
             | finite time. (Not the best definition, I know, but bear
             | with me for a second).
             | 
             | If you reverse the _successor_ operator, you get the
             | _predecessor_, which can be used to dismount large numbers
             | and make them smaller. If you apply _predecessor_ to '1',
             | you get 'predecessor(1)' which doesn't match any of the
             | *natural* numbers as defined in the paragraph above.
             | However, this new number is useful because you can map it
             | to a collection without elements, allowing you to count a
             | set of size 'zero'. This set doesn't exist, but it has a
             | well defined size thanks to the process explained above.
             | (You may as well apply _predecessor_ to 'zero' and count
             | *negative numbers*, which allow you to create a mapping
             | with debt, and thus count _I owe you_ amounts that don't
             | exist in the physical world either).
             | 
             | Now as for real and complex numbers, they can't count
             | physical objects as you said because at some point you
             | can't measure smaller and smaller magnitudes (though you
             | _can_ create orders with them, even if it doesn 't directly
             | involve the _successor_ operator). But mathematicians use
             | them to count the size of _infinite sets_, which being
             | immaterial never lose precision; you just know you can
             | repeat their defining processes once and again, showing
             | that there exist a mapping between any step in the process
             | and the instance of the real or complex number that
             | correspond to its size.
        
             | HelloNurse wrote:
             | In practical terms, "counting and measuring" means well-
             | behaved arithmetic operations like addition, subtraction,
             | multiplication, division, roots etc. (and often specific
             | algebraic structures like rings, fields etc.)
             | 
             | Rational and real numbers represent the most intuitive
             | concept of quantity with different cardinality; natural
             | numbers are more basic in theory but a restricted special
             | case in most application (they can only count "countable"
             | objects and they aren't dense); complex numbers compensate
             | unnatural weirdness with important applications; then there
             | are other very number-like niches (e.g. p-adic numbers,
             | quaternions, algebraic extension fields...) and number-
             | based arbitrary constructions (multidimensional vector
             | spaces, matrices and tensors...).
             | 
             | > Show me how you can count with reals.
             | 
             | Like you count with natural numbers, plus many other
             | possibilities because there are more numbers to count with.
             | The same applies to rational numbers (which exist more than
             | real numbers), complex numbers, etc.
        
               | TuringTest wrote:
               | > The same applies to rational numbers (which exist more
               | than real numbers)
               | 
               | Wait, did you say there are more rationals than reals?
               | Isn't that the other way around?
               | 
               | I don't know if that's a slip of the tongue or I'm
               | missing something in my recall of basic math lessons
        
               | gowld wrote:
               | They are "more existent" ("less fictional"), not "more in
               | cardinality".
        
               | TuringTest wrote:
               | How are they more existent? They are both patterns of
               | recognized concepts in human brains.
               | 
               | Is it because more people understand rationals than
               | reals? Ok, that would make rationals more existent, as
               | they exist in more brains.
        
               | HelloNurse wrote:
               | Real numbers do not exist: they are convenient, but there
               | is a high risk of crossing over from reality-relevant
               | math to nonsense because of infinite calculations.
               | 
               | Real numbers are of course more numerous than rational
               | numbers, just like unicorn horns are more numerous than
               | horse horns, but it's an entirely different question.
        
               | TuringTest wrote:
               | No numbers exist, they are abstraction on human-created
               | procedures. Even naturals, which are an abstraction over
               | pointing to things and counting.
        
       | Decker87 wrote:
       | Honest question, what is the usefulness of proving such an
       | obvious thing?
        
       | question000 wrote:
       | The only thing this proves is that mathematics is a soft science,
       | where concepts like "number" and "infinite" are subjective.
       | 
       | There are obviously infinite numbers, if you think there's a
       | finite number of numbers, take that number and add one to that.
       | QED
        
         | dang wrote:
         | " _Please don 't post shallow dismissals, especially of other
         | people's work. A good critical comment teaches us something._"
         | 
         | https://news.ycombinator.com/newsguidelines.html
        
           | question000 wrote:
           | I can't believe you called this shallow and didn't even
           | include an explanation as to why, ironic. There's nothing
           | shallow about this at all, it goes to the heart of this fake
           | intellectualism on this site.
        
             | dang wrote:
             | It's shallow because (a) contentless denunciations of "soft
             | science" are cliche; (b) reducing serious mathematical work
             | to "obvious" is the worst sort of dismissal.
             | 
             | Please don't post like this to HN. We're trying for higher-
             | quality discussion.
             | 
             | https://news.ycombinator.com/newsguidelines.html
        
               | question000 wrote:
               | Seriously fuck off, everyday I read the same bullshit
               | like this and it never gets removed. Read your own
               | comments! This is laughable.
               | 
               | Some things are obvious, and I'm allowed to say that to
               | these self-proclaimed mathematicians.
        
         | creddit wrote:
         | 1) Mathematics is in no way a science.
         | 
         | 2) I think you didn't read the article at all.
        
           | question000 wrote:
           | Yeah of course I didn't read the article. It's literally was
           | the same as an article saying "Cows are green! If you're
           | smart enough it becomes math!" I can't feed the clickbait
           | machine.
        
       | kibwen wrote:
       | _> Woodin named the axiom (*), pronounced "star," because it was
       | "like a bright source -- a source of structure, a source of
       | light," he told me._
       | 
       | There are only two hard problems in mathematics: infinity and
       | naming things.
        
       | ProfHewitt wrote:
       | The results in the article are based on 1st-order logic,
       | 
       | which is inadequate for the foundations of mathematics.
       | 
       | Mathematical abstractions need to be characterized up a
       | 
       | unique isomorphism as in the theory Ordinals described in
       | 
       | the following article:                   "Theory Ordinals can
       | Replace ZFC in Computer Science"
       | https://papers.ssrn.com/abstract=3457802
       | 
       | Mathematical questions can be properly addressed only by
       | 
       | using adequate foundations.
        
       | sharpener wrote:
       | The mileage of others may vary, but it has been my experience
       | that there is no cogent solid proof of uncountability that can
       | withstand concerted critique.[0]
       | 
       | Being charitable one might argue that the meanings of terminology
       | had been lost in translation over time and that perhaps Cantor
       | was trying to create non-standard analysis, but then the diagonal
       | argument seems to represent nothing more than the truism that
       | finite numbers are smaller than transfinite ones.
       | 
       | Hence I worry about people who are still worrying about this
       | issue, and I worry for the future of science and AI in particular
       | if folks can't get clear of it.
       | 
       | [0]
       | https://www.researchgate.net/publication/328568169_The_Case_...
       | 
       | Yes, I'm that guy who wrote that.
        
         | myWindoonn wrote:
         | Hi, Lawvere pummelled your position into the ground a while
         | ago: http://tac.mta.ca/tac/reprints/articles/15/tr15.pdf Your
         | critique involves repeatedly crossing the boundary between the
         | inside and outside of the system in question; Lawvere works
         | entirely inside the system, and shows that the paradoxes of
         | self-reference arise from our interpretations.
         | https://arxiv.org/abs/math/0305282v1 explains with many
         | examples.
         | 
         | Hi downvoters: Use your words when somebody is wrong. Your
         | downvotes aren't helpful here for finding the truth.
        
           | sharpener wrote:
           | Lawvere's paper appears to suggest that if I refute one
           | diagonal argument then I refute them all. Would you consider
           | that to be an accurate description?
        
           | sharpener wrote:
           | I'm not seeing this yet. Perhaps you could explain it to me
           | in plain terms.
        
         | curtisf wrote:
         | This presents a confused understanding of Cantor's
         | diagonalization argument.
         | 
         | You are shrouding in complexity something that is
         | straightforward. The complete proof of distinct infinite
         | cardinalities can be stated succinctly and clearly in only a
         | few lines, without referencing the reals at all. You don't need
         | to vaguely refer to "four steps", you should precisely
         | elaborate the steps of the proof you view as problematic.
         | 
         | First, forget the reals. Let's establish that there are sets
         | that are infinite yet uncountable.
         | 
         | ----
         | 
         | "Uncountable" means that there is no bijection with (a subset
         | of) the natural numbers.
         | 
         | An "infinite sequence of A's" means a function N - A. For
         | convenience, I'll denote the set of infinite sequences of A's
         | with `A**` -- this is not standard notation.
         | 
         | Theorem: There is no bijection between N and {0, 1}**.
         | 
         | Proof: Suppose for contradiction that there is a bijection f: N
         | - {0,1}**.
         | 
         | Define a function z(n) = 1 - f(n)(n).
         | 
         | z is clearly well defined and is clearly a member of {0,1}**.
         | 
         | Since f is a bijection and z is in the codomain of f, there is
         | an element k of N such that f(k) = z.
         | 
         | However, z(k) = 1 - f(k)(k). Yet f(k) = z, so z(k) = 1 - z(k).
         | 
         | This is a contradiction. So, the assumption (that there exists
         | a bijection f: N - {0,1}**) must be false.
         | 
         | Q.E.D.: There does not exist a bijection between N and {0,
         | 1}**.
         | 
         | ----
         | 
         | The linked paper claims that the above proof would work on a
         | bijection of the form N - [N] or N - [Q] (where the [] indicate
         | some suitable representation of the set as sequences of bits;
         | again, not standard notation), but that is mistaken: crucially,
         | the constructed function `z` must be a member of the codomain
         | of `f`. The construction 1 - f(n)(n) does not necessarily lie
         | in that set when the set only permits _certain_ bit-sequences!
         | 
         | For example, consider an encoding [N] which is "one-hot"; the
         | natural n is encoded as a function y(n) = 1 and otherwise y(k)
         | = 0. Then there is an obvious bijection `f`: f(a, b) = 1 when
         | a=b and otherwise 0. What is `z` for such an `f`? z(k) = 1 -
         | f(k, k) = 1. So, `z` is not a member of [N].
         | 
         | For a more involved example, consider an encoding of [NN] that
         | is a unary-encoding of the first natural, followed by a single
         | zero, followed by a unary encoding of the second natural, and
         | followed by only 0s. (The rationals correspond to a subset of
         | these). A standard bijection has the pairs appear in the order
         | of their sum: (0, 0), (0, 1), (1, 0), (2, 0), (1, 1), (0, 2),
         | ... It's an exercise left for the reader that z constructed on
         | this set does not encode to a unary number, followed by a
         | single zero, followed by a unary number, followed by only
         | zeros.
         | 
         | ----
         | 
         | If you have an issue with cardinalities, I expect it can be
         | found in the above proof and does not actually lie with the
         | reals. But, for completion, to tie in the real numbers, it's
         | necessary to show that {0,1}** is bijective with R, or with (0,
         | 1). This falls out from the Cauchy-sequence definition of R.
         | For a sequence y : {0,1}** the sequence z(k) = y(0) + 1/2*y(1)
         | + 1/4*y(2) + 1/8*y(3) + ... + 1/2^k*y(k) is clearly a Cauchy
         | sequence with a limit in the interval [0, 1]. It's also
         | straightforward to check that every real number in the interval
         | [0, 1] is the limit of one such Cauchy sequence.
        
           | sharpener wrote:
           | > Define a function z(n) = 1 - f(n)(n).
           | 
           | I don't understand the notation f(n)(n). Is it related to
           | f_{nn} in LaTeX notation? Your later text suggests maybe it
           | was aiming at f(n,n) so I will assume that.
           | 
           | I recognise a form of this argument and I might have tackled
           | it in the supplementary materials I created that are
           | referenced in the article. Let me know.
           | 
           | > However, z(k) = 1 - f(k)(k). Yet f(k) = z, so z(k) = 1 -
           | z(k).
           | 
           | I'm assuming this was intended to be: z(k) = 1 - f(k). Yet
           | f(k) = z, so z(k) = 1 - z(k).
           | 
           | For some k, z(k) = 0.5. f(k) = 0.5. Seems Ok.
        
             | drdeca wrote:
             | For each natural number k, f(k) is itself a function, and
             | f(k)(k) means the value of that function at k.
             | 
             | Yes, you could basically think of it as f_{k,k} if you
             | wanted to.
             | 
             | No, this is __not__ meant to be 1 - f(k) . f(k) is a
             | function (or a sequence, if you prefer), not a particular
             | value in {0,1}, f(k)(k) is a particular value in {0,1}.
             | 
             | 0.5 is not in the set {0,1}, and therefore if z(k)=0.5 then
             | z is not in {0,1}* .
        
             | myWindoonn wrote:
             | For future readers: z cannot return 0.5. z can only return
             | 0 or 1. This is because z is closed over a function which
             | returns 0 or 1, and inverts it to return 1 or 0.
             | 
             | This should remind folks of both Turing's Halting problem
             | and Russell's paradox. z takes some f which claims to be a
             | bijection (claims to Halt, claims to be a set of all sets)
             | and finds a way to call f against a witness constructed
             | from f.
        
           | sharpener wrote:
           | > Theorem: There is no bijection between N and {0, 1}*.
           | 
           | So no bijection between \bb{N} in an unspecified number base
           | and some binary strings that could represent integers? That's
           | a nonstarter for me.
           | 
           | Please read my article. Thanks.
        
             | myWindoonn wrote:
             | You misread their type. There is no bijection N -> (N ->
             | 2); {0,1}* = 2* = N -> 2.
             | 
             | Note that the given proof is a version of Lawvere's fixed-
             | point theorem. The trick is noticing that for some x : N, f
             | : N -> (N -> 2) can be applied onto it twice, giving f(x) :
             | N -> 2 and f(x)(x) : 2.
             | 
             | Note further that infinite binary strings don't just
             | represent natural numbers. Indeed, every natural number has
             | a _finite_ binary string, and that is the bijection that
             | you 're imagining. The question is, which natural number is
             | represented by 111...? This leads to the difference between
             | the natural numbers, their one-point compactification, and
             | Cantor space in terms of searchability. Escardo has an
             | article on this:
             | http://math.andrej.com/2007/09/28/seemingly-impossible-
             | funct...
             | 
             | Please read Lawvere's article. Thanks.
        
         | candybar wrote:
         | > Stage 4 of the CDA outline can be critiqued on several
         | grounds. The first is that if we have all the numbers in [0,1]
         | in our table a priori, then using the diagonal process to
         | create a number that is not in the original set does precisely
         | that: It creates a number that is not in [0,1]. So why does the
         | constructed anti-diagonal number seem to have a form that puts
         | it within [0,1]? Could it be that ignoring the zero ahead of
         | the decimal point is a bit sneaky?
         | 
         | You wrote all this and you don't seem to understand how proof
         | by contradiction works?
         | 
         | > If, instead, we construct the anti-diagonal number by
         | counting down the table by positions before each digit
         | selection we find that any anti-diagonal number constructed
         | (from the rectangle) is now within the set we have counted
         | through, and therefore not outside the counted set. This
         | Stretched Diagonal counterargument remains true as d- [?],
         | irrespective of any ordering of number representations in the
         | table.
         | 
         | This is not how anything works. You can't take a proof where
         | they create an example that leads to contradictions and say,
         | well, if you choose a different example, it doesn't lead to
         | contradictions, so the proof doesn't work.
         | 
         | Your "proof" in Appendix B is also just gibberish from a
         | mathematical perspective. The infinity and cardinalities aren't
         | numbers - this isn't a mathematically rigorous statement: lim(n
         | -> inf) log(n) = inf = aleph_0 - it's nonsense. What you're
         | saying here is that as n increases without bound, 2^n also
         | increases without bound. That doesn't prove anything about the
         | cardinality of infinite sets.
         | 
         | Also, since you seem to accept that |R| is equivalent 2^|N|,
         | it's also not hard to prove that 2^|S| > |S| for any S. 2^|S| <
         | |S| is trivially untrue and if |S| = 2^|S], there must be at
         | least one bijection F such that F: S => P(S) and inverse F^-1:
         | P(S) => S
         | 
         | Now define S' to be a subset of S such that s is in S if & only
         | if it's in S, but not in F(s). Now, consider F^-1(S') - it must
         | be in S, due to how it's defined. Let's consider whether it's
         | also in S':
         | 
         | 1. If F^-1(S') is not in S' (= F(F^-1(S')), by definition, it
         | must be in S'
         | 
         | 2. If F^-1(S') is in S', by definition it must not be in S'
         | 
         | Since both these lead to contradictions, the assumption that
         | there must be a bijection between P(S) and S mut be false.
         | 
         | On the whole, you seem to be generally confused - R is not a
         | real thing - it's a mathematical abstraction.
        
       | xvilka wrote:
       | There is a good video about that on Numberfile[1]
       | 
       | [1] https://www.youtube.com/watch?v=5TkIe60y2GI
        
       | joe_the_user wrote:
       | _For 50 years, mathematicians have believed that the total number
       | of real numbers is unknowable._
       | 
       | It's an established result that the Continuum Hypothesis is
       | independent of Zermelo-Fraenkel axioms of set theory. No proof is
       | going change that.
       | 
       | So whatever has been proved here doesn't change that. It will
       | take a second to get the reference but Raymond Smullyan says
       | essentially that "we're not looking for a proof or disproof of
       | CH, we're looking for an assumption that can be shown to be
       | natural enough that we can take it as an axiom".
       | 
       | Just sayin' since the (subheading) writer seems to be playing
       | fast and loose with the concepts involved.
       | 
       | Edit: article goes on to give rigorous explanation but still
       | starting "no one knew" confuses what's going on.
        
       | js8 wrote:
       | I have a question, so assuming that we adopt the axiom(s) that
       | cause real numbers to be cardinality Aleph_2 and not Aleph_1, is
       | then there some intuitive explanation how does the set of size
       | Aleph_1 looks like, and what it could be used for?
        
       | tromp wrote:
       | The explanation of Cohen's filter for making new reals reminds me
       | of surreal numbers {L | R} but in this case every real is in
       | either L or R. Still, I fail to see how that gives you more than
       | restricting L and R to contain only rational numbers...
        
       | mmmBacon wrote:
       | Maybe I misunderstood the article but if the set of real numbers
       | is finite then it should be countable. But I can easily prove
       | that the set of real numbers or any subset of real numbers is not
       | countable. Been a really long time since I've thought about this
       | but wondering what I'm missing.
        
         | TameAntelope wrote:
         | How can you easily prove that the set of real numbers is not
         | countable? I don't think it's as easy as you claim, but I'm
         | kind of a dummy so it's quite probably I'm wrong.
        
           | twelfthnight wrote:
           | Here's some background on the proof [1]. Here's a video
           | explaining it little better [2].
           | 
           | [1]
           | https://en.wikipedia.org/wiki/Cantor%27s_diagonal_argument
           | [2] https://www.youtube.com/watch?v=elvOZm0d4H0
        
             | TameAntelope wrote:
             | I guess I was focusing on the word "easy", heh.
        
               | mmmBacon wrote:
               | You do this in sophomore level Real Analysis. So as far
               | as pure math goes, easy.
        
               | TameAntelope wrote:
               | > sophomore level Real Analysis
               | 
               | Hmm?
        
               | rpearl wrote:
               | We did this first semester freshman year at my school in
               | a single lecture in a 100 level "concepts of mathematics"
               | course. I get that there's a certain amount of "it's hard
               | to wrap your head around infinity" but really this proof
               | is pretty well trodden by students not far into learning
               | mathematics.
        
               | TameAntelope wrote:
               | Which school?
        
               | doctoboggan wrote:
               | Watch the video, its actually quite easy to follow.
        
         | Sharlin wrote:
         | No, what the article is talking about is the question whether
         | or not the cardinality of real numbers is the _smallest
         | uncountable infinity_ or some other, larger uncountable
         | infinity. The only _countable_ infinity is aleph-0, the
         | cardinality of natural numbers, and Cantor showed that aleph-0
         | is too small to hold all reals. So reals must be uncountable,
         | but there is an infinite hierarchy of uncountable infinities,
         | and it is not known which one is the cardinality of reals
         | (although in practice it 's suspected to be either aleph-1,
         | which is what the Continuum Hypothesis states, or aleph-2).
        
           | orangecat wrote:
           | _(although in practice it 's suspected to be either aleph-1,
           | which is what the Continuum Hypothesis states, or aleph-2)_
           | 
           | There are some arguments that it's "really" much larger, as
           | in greater than aleph-n for any finite n. See
           | https://risingentropy.com/the-continuum-hypothesis-is-false/
           | (incidentally an excellent blog if you're interested in these
           | sorts of things).
        
           | keymone wrote:
           | unrelated: how do we know there are no alephs between 0 and
           | 1?
        
             | elcomet wrote:
             | That's the definition of aleph-1.
             | 
             | The continuum hypothesis is that 2^aleph-0 (number of sets
             | of integers or number of reals) = aleph-1.
        
             | Sharlin wrote:
             | If you assume Axiom of Choice, you can define cardinalities
             | in terms of ordinal numbers (an extension of naturals that
             | generalizes the notion of "counting", or "indexing"). And
             | ordinal numbers are well-ordered, ie. every element has a
             | unique "successor" element.
        
       | jostmey wrote:
       | > "Not all infinities are equal"
       | 
       | In other words, there are different categories of infinite, and
       | it might be inappropriate to represent infinity with just one
       | symbol! This article is about how many types of infinity might
       | exist.
       | 
       | I was taught there is countably and uncountably infinite.
       | Integers are countably infinite because the number of integers
       | between any two numbers if finite. Real numbers are uncountably
       | infinite because there are infinite numbers between any two real
       | numbers.
        
         | ceh123 wrote:
         | >I was taught there is countably and uncountably infinite.
         | Integers are countably infinite because the number of integers
         | between any two numbers if finite. Real numbers are uncountably
         | infinite because there are infinite numbers between any two
         | real numbers.
         | 
         | So this isn't exactly right, although I suppose the argument
         | for integers isn't exactly wrong. When we get to the rational
         | numbers however, they are countable but there are an infinite
         | number of rational numbers between any two rationals in the
         | typical way of thinking about "between" numbers. Based on your
         | argument above, this would mean that rationals are uncountable.
         | 
         | A better way to think about this is that the set of natural
         | numbers is the first (and smallest) infinite set you can
         | construct. This is the set of all counting numbers so we call
         | it countable.
         | 
         | We then say two sets are the same size (cardinality) if you can
         | create a one-to-one mapping between the two that covers both
         | sets (a bijection). This way you exactly pair one element of
         | one set with one element of another. You can do this with the
         | natural numbers and integers by just alternating positive and
         | negative (so 0 -> 0, 1 -> 1, 2 -> -1, 3 -> 2, etc.). All the
         | sets that you can do this sort of mapping with the natural
         | numbers are considered countable.
         | 
         | You can't do this sort of mapping from the natural numbers to
         | the reals (see cantor's diagonalization argument) so since you
         | can't "count" the reals, the set is "uncountable."
        
         | omnicognate wrote:
         | The article isn't about how many types - or rather
         | cardinalities (sizes) - of infinity exist, it's about which of
         | those cardinalities describes the real numbers.
         | 
         | "Countably" infinite sets have cardinality Aleph_0, the
         | "smallest infinite size". There is an infinity of "larger
         | infinite sizes", all of which are "uncountable", so to refer to
         | a set as "uncountably infinite" doesn't pin down which
         | particular cardinality it has.
         | 
         | The article is about what specific cardinality the set of real
         | numbers has, and in particular whether it's Aleph_1 or Aleph_2
         | as it seems less likely to be any of the infinite other
         | possibilities.
        
         | jmgao wrote:
         | > I was taught there is countably and uncountably infinite.
         | Integers are countably infinite because the number of integers
         | between any two numbers if finite. Real numbers are uncountably
         | infinite because there are infinite numbers between any two
         | real numbers.
         | 
         | This is unsound: a set is countable if there's a mapping that
         | assigns an integer to each element of that set. There are an
         | infinite number of rational numbers between any pair of
         | rational numbers, but you can pretty easily construct a mapping
         | between integers and rationals. Imagine representing a rational
         | x/y as a point on the plane, and draw a square spiral around
         | the origin. Every rational number lays on that spiral, and
         | there's a unique "shortest arc-length of the spiral from the
         | origin" for each rational.
        
           | gotostatement wrote:
           | I guess you could rephrase OP's argument in a sound way as: a
           | set is countable if there's a way to order it such that
           | between every two numbers there are finitely many numbers
        
         | chaoticmass wrote:
         | https://www.youtube.com/watch?v=SrU9YDoXE88
         | 
         | I really like how VSauce covers this topic as well.
        
         | mikepurvis wrote:
         | Veritasium did a really good video on this recently, based
         | around Hilbert's Hotel:
         | 
         | https://www.youtube.com/watch?v=OxGsU8oIWjY
        
       | ncmncm wrote:
       | > _How many real numbers exist?_
       | 
       | You might be tempted to say "lots". And you would be right, as
       | far as that goes.
       | 
       | But that doesn't satisfy a _real_ mathematician. The question
       | that immediately arises is whether  "lots" is "enough". And that
       | leads the better sort of mathematician inevitably to: "enough for
       | what?" That is what mathematicians are deep in the middle of
       | exploring, now.
       | 
       | For example, when you are asked, "Does this skirt make my butt
       | look too big?", you obviously must not say "yes", but you just as
       | obviously cannot, with an entirely clear conscience, say "no",
       | either. But for anyone with an intact survival instinct, the
       | counter-question, "too big for what?" should spring immediately
       | to mind. And it's a good one, but it depends intimately on the
       | true size of the set of real numbers. So, this is not an idle
       | pursuit.
        
         | rkagerer wrote:
         | _" Does this skirt make my butt look too big?"
         | 
         | "Too big for what?"_
         | 
         | Yeah let me know how that works out.
        
           | philipswood wrote:
           | Yes, the call for more experimentation to keep theorists
           | honest.
        
         | 90minuteAPI wrote:
         | Answering "Does this skirt make my butt look too big?" with
         | something that "depends intimately on the true size of the set
         | of real numbers" seems to be among the worst strategies.
        
           | ncmncm wrote:
           | The mathematics of infinities are nothing if not
           | counterintuitive.
        
         | whoaisme wrote:
         | Ken M is that you?
        
       | dexwiz wrote:
       | I'm no mathematician, but I have always found it strange
       | infinities are talked about as physical states (towers of tall
       | towers, etc), and not functions.
       | 
       | Integer number counting is essentially a successor function; take
       | N, add 1, output N+1. One input, one output.
       | 
       | Real number counting is a bit more loose. To find the numbers
       | between .1 and .2, you find all fractionals of a given size,
       | normally 1/10. So we get .10, .11, .12, etc. This function gives
       | more outputs than inputs, thus an increase in cardinality.
       | Forcing just seems like changing the function to increase the
       | cardinality.
       | 
       | In my mind, all infinities are equal, it's how you generate them
       | that matters.
        
         | drdeca wrote:
         | There is a way of mapping different ordinal numbers which are
         | less than some large countable ordinal (idr how high up it
         | goes. I suspect it isn't well defined at least past the church
         | kleene ordinal, but I think it is much before that, but also I
         | could be totally wrong) to different functions from integers to
         | integers which increase at different rates.
         | 
         | These methods include the Hardy hierarchy, and the fast growing
         | hierarchy.
         | 
         | (But regarding all infinities being equal, that's just
         | incorrect, sorry.)
        
         | zeugmasyllepsis wrote:
         | > Real number counting is a bit more loose. To find the numbers
         | between .1 and .2, you find all fractionals of a given size,
         | normally 1/10.
         | 
         | Unfortunately the fractionals method does not work, since there
         | are numbers (in fact, uncountably infinitely many!) irrational
         | numbers which cannot be expressed as fractions, which are
         | nonetheless real numbers (such as the square root of two and
         | pi).
        
         | IncRnd wrote:
         | I'm not a mathematician, either, but I believe your comment
         | shows why not all infinities are equal.
         | 
         | When you attempt to enumerate .1, .2, .3, .4, etc. you get an
         | infinite count of numbers. However, when you perform that same
         | operation on .05, .15, .2, .25, and so on, you will get "twice
         | as many numbers" as the first time around.
         | 
         | In this way, the second infinite sequence has "twice as many
         | numbers" as the first infinite sequence.
        
           | gowld wrote:
           | No, it doesn't. Then have the same number, as can be plainly
           | seen by lining them up and counting the elements in order.
        
       | arduinomancer wrote:
       | I don't get for Cantor's diagonalization proof, why do we need to
       | use the diagonal digits to form the new number?
       | 
       | Would the proof work the same if we instead used the first digit
       | of every number in the list?
        
         | pfortuny wrote:
         | Then you cannot guarantee that when you change the first digit
         | of the second number, what you get is not the first number.
        
         | ceh123 wrote:
         | To give a concrete example for you, lets start with the finite
         | set:
         | 
         | {0.22, 0.32, 0.33}
         | 
         | We now construct a new number in this way:
         | 
         | First digit of the first number is 2, so we take 3 which is
         | different as our first digit.
         | 
         | First digit of the second number is 3 so we take 2 as our
         | second number. Now we have 0.32
         | 
         | First digit of the third number is 3 so we take 0 as our third
         | number.
         | 
         | We've constructed the number: 0.320 = 0.32 which is in the set
         | already. So this construction method doesn't guarantee that
         | it's a new number, only that it's different from the first.
         | 
         | With Cantor's Diagonalization argument we guarantee it's
         | different from the first number since it's different in the
         | first digit, then we guarantee it's different from the second
         | since it's different in the second digit, etc. etc. it's
         | different from all the numbers in our list.
         | 
         | To take the same set above as an example, we end up creating a
         | number like: 0.318 which is definitely different from the
         | numbers in the set
        
         | miloignis wrote:
         | There are only 10 possible first digits of the new number, so
         | you can't choose a number that will differ from all other
         | numbers in the first digit. If you do it diagonally, you'll
         | always have 9 other options to choose from, since you just have
         | to make it different from that one number!
        
           | arduinomancer wrote:
           | Makes sense, thanks
        
         | tpetrina wrote:
         | That wouldn't work because n-th digit differing from 1st digit
         | of n-th number doesn't guarantee that the new number isn't
         | accounted for already.
        
         | erdos4d wrote:
         | You need the diagonal because you are making a new number in
         | (0,1) that can't be in the list by just choosing the nth digit
         | of said number different than the nth digit of the diagonal.
        
         | pavpanchekha wrote:
         | To fill in some details:
         | 
         | Two real numbers are different if in the same place they have
         | different digits. [1] So the idea is to take your infinite,
         | hypothetical list of all real numbers A[] and make a new real
         | number X, where for all i, there's some digit where A[i] and X
         | differ. Easiest way is to make X differ from A[i] at digit i.
         | 
         | [1] There's a subtlety here about repeating nines at the end of
         | a number, but it is inessential.
        
       | chasing wrote:
       | There are exactly 1,837 real numbers and anyone who tells you any
       | different is just trying to sell you something.
        
       | privatdozent wrote:
       | Another great piece from Quanta. Such an important institution
        
       | btilly wrote:
       | As a constructivist I'll be over in the corner that says that
       | there are only a countable number of real numbers, and the
       | unimaginable number of unimaginable infinities that classical
       | mathematics insiste exists is all made up nonsense. That, in
       | fact, things that can't ever be named, even in principle, don't
       | actually exist.
       | 
       | What is interesting is that as shocking as constructivism may be,
       | there is no logical flaw in it. If classical mathematics is
       | consistent, then so is constructivism. And "there exists" is a
       | whole lot more meaningful in constructivism.
        
         | papandada wrote:
         | I have found my people. Thank you.
        
         | reggieband wrote:
         | I'd like to subscribe to this newsletter.
         | 
         | Mathematicians using sets to argue about the size of
         | uncountably infinite Reals feels like the old joke of
         | philosophers arguing over how many angels can fit on the head
         | of a pin. I often wonder if we've used set theory (and
         | formalisms of math based on logic) to substitute one
         | metaphysical theory for another equivalent one.
         | 
         | I can understand the frustration of mathematicians of the past
         | being told they have to ensure their theories align with some
         | spiritual mythology. But it sort of feels like I'm being forced
         | to accept some kind of metaphysical truth in the name of
         | ensuring the calculus of infinitesimals is grounded in logic.
        
         | scapp wrote:
         | You sound like you need to read this [0] answer to the question
         | "Are real numbers countable in constructive mathematics?".
         | 
         | > You are using the word "constructive" in an unusual way. It
         | is true that, in ZFC, the set of computable real numbers is
         | countable, but that is not directly a statement about
         | constructive mathematics.
         | 
         | > Not every school of constructive mathematics identifies real
         | numbers with algorithms; that's a characteristic of the
         | "Russian" school as I understand it. In other schools of
         | constructivism that I am more familiar with, real numbers are
         | coded by elements of 2^o, or by a certain type of Dedekind cut
         | on the rationals. In such schools it is not universally assumed
         | that every real number is associated with a finite algorithm.
         | 
         | > Even in the Russian school, they would not say that the set
         | of real numbers is countable. Because, if you identify real
         | numbers with algorithms, there is no computable enumeration of
         | computable reals that lists all computable reals, and so the
         | translation of "the real numbers are countable" into this
         | setting is false.
         | 
         | > That phenomenon also occurs in classical computable analysis.
         | The subsystem RCA0 of second-order arithmetic has a model in
         | which every real number is computable. But this subsystem still
         | proves that there is no surjection N -> R.
         | 
         | ---
         | 
         | It's worth noting that another answer to that question (Andrej
         | Bauer's) gives a constructive proof that the real numbers are
         | uncountable using with the axiom of countable choice. The
         | question of whether it's provable without the axiom of
         | countable choice is open (though see this [1] fairly recent
         | paper on a proof for MacNeille reals).
         | 
         | [0] https://mathoverflow.net/questions/30643/are-real-numbers-
         | co...
         | 
         | [1] https://arxiv.org/abs/1902.07366
        
           | btilly wrote:
           | Yes, there are multiple constructivist approaches possible.
           | However since my objection to classical approaches is that I
           | want "X exists" to be meaningful, I like mathematical objects
           | that can be written down with a finite number of symbols in a
           | finite space. Which means that I'm only interested in a
           | countable universe of possible mathematical things.
           | 
           | If you say "exists" about anything else, I'll understand you
           | - I do have advanced degrees in math. But I'll think that
           | you're using the word "exists" in a deeply artificial way.
        
             | pdonis wrote:
             | _> I like mathematical objects that can be written down
             | with a finite number of symbols in a finite space._
             | 
             | Which is fine, but I don't think it justifies the claim
             | that there are only countably many real numbers. The only
             | claim it justifies is that there are only a finite (not
             | even countable, since "countable" implies infinitely many)
             | set of numbers that you find useful.
        
             | LudwigNagasena wrote:
             | What's the problem with it being "artificial"? Is your
             | problem purely linguistic? You just dislike the word
             | "exists" being used in this context?
        
               | btilly wrote:
               | The problem is that exists comes to mean something
               | technical that doesn't match common usage.
               | 
               | Let's take my favorite example.
               | 
               | In graph theory, a minor of a graph is a graph you can
               | get by removing vertices, removing edges, or by replacing
               | an edge-vertex-edge triple with a single edge. Many
               | categories of graphs are closed under the act of taking
               | minors. For example planar graphs, graphs you can draw on
               | the plane with no crossings, are.
               | 
               | The category of planar graphs is entirely described by
               | the fact that any graph that isn't planar must have
               | either K5 or K3,3 as minors. That is, a graph with 5
               | vertices, all connected. Or a graph with 2 groups of 3
               | vertices, that all connect to each other. Therefore we
               | call those two graphs the "forbidden minors" for planar
               | graphs.
               | 
               | The Robertson-Seymour theorem says that any category of
               | graphs which is closed under graph minors has a similar
               | description. There is a finite list of forbidden minors
               | which, if none are minors of a given graph, then that
               | graph is in the category.
               | 
               | Since there is a polynomial time algorithm to detect
               | whether a given graph is a minor of another, this
               | immediately means any category of graphs closed under
               | taking minors must have a polynomial time algorithm to
               | test for membership. Just test each forbidden minor.
               | 
               | So far this is straightforward, but here is where things
               | get weird.
               | 
               | The first catch is that the Robertson-Seymour theorem is
               | non-constructive. That is, it says that the list exists
               | and is finite. But it does not bound the number. It does
               | not give us a way to find those minors. It does not give
               | us any way to determine whether we have a complete list.
               | For example we know of thousands of minimal forbidden
               | minors for graphs that can be drawn on a torus, and do
               | not know if our list is complete.
               | 
               | The second catch is that we know that none of those
               | things are possible to do. That is, there are collections
               | of categories of such graphs such that we can prove that
               | no algorithm can bound the number, no algorithm can
               | search for those examples, and no algorithm can verify
               | that a list of forbidden minors is complete.
               | 
               | In what sense does a finite thing that is unfindable,
               | unverifiable, and of unknowable size actually exist? And,
               | if you think that it exists, in what sense is something
               | of unboundable size actually finite?
        
               | LudwigNagasena wrote:
               | In the usual sense. I don't see a problem. Just because
               | you don't have a perfect knowledge of something, it
               | doesn't mean that the thing isn't real.
               | 
               | I am not sure where this idea even comes from? To be
               | honest, this sounds completely ridiculous.
        
               | pdonis wrote:
               | These concerns don't apply to the claim that the set of
               | real numbers is uncountable. Cantor's diagonal proof is
               | constructive: given any countable set of real numbers, it
               | tells you how to construct a real number that is not in
               | the set. That is sufficient to show that the set of real
               | numbers cannot be countable. Also, even though many real
               | numbers cannot be written down with a finite set of
               | symbols, Cantor's diagonal proof can be.
        
               | btilly wrote:
               | Yes, you can write down Cantor's diagonal proof. But if
               | you're careful about it, you find that the diagonalized
               | thing is not a real number. For example a program to list
               | programs that can be proven (by some set of axioms) to
               | create Cauchy sequences can be used to create a Cauchy
               | sequence, but you can't _prove_ that that sequence is a
               | Cauchy sequence without running into Godel 's
               | incompleteness theorem.
        
         | chmod775 wrote:
         | > there are only a countable number of real numbers
         | 
         | Then you should be able to come up with a function that assigns
         | a natural number uniquely to each real number.
         | 
         | Of course if you tried that I could immediately name you a real
         | number, or a pair of them, for which your rule doesn't work.
        
           | myWindoonn wrote:
           | Turing machines are countable. In some constructive settings,
           | every real number is computable, as an axiom; those settings
           | allow us to talk about the Turing machines which compute a
           | particular real number.
        
           | skoodge wrote:
           | That depends on what you mean by "assigns uniquely", "rule"
           | and "doesn't work", which is why this question is deeply
           | entangled with philosophical issues that cannot be settled
           | purely mathematically.
           | 
           | It is obvious that all expressions in the English language
           | can be ordered from smallest to largest and
           | lexicographically, which makes these expressions trivially
           | countable. We can thus assign natural numbers to real numbers
           | by assigning numbers to their expressions in a natural or
           | formal language, which will of course include infinitely many
           | expressions that are just nonsense descriptions and
           | infinitely many expressions that map to the same real number.
           | These expressions will also include any possible expressions
           | of Cantor's or other diagonalized numbers. In such a sense
           | then, we can trivially "count" the real numbers unless we
           | hold the philosophical view that there are real numbers that
           | are not expressible. This is where it becomes a question of
           | philosophy of mathematics, not mathematics proper.
           | 
           | You can of course object that what you meant by "assigns
           | uniquely" is an _unambiguous 1:1 mapping_ and that including
           | any number of nonsense descriptions misses the point. In that
           | case giving a  "rule doesn't work" because the diagonalized
           | number always escapes the proposed system of counting the
           | numbers, but only because the diagonalized number is allowed
           | to 'parasitically' depend on the totality of the system, but
           | is excluded from the system (or else it would diagonalize
           | itself and become ambiguous at that particular decimal
           | place). This particular viewpoint is tied to a particular
           | philosophical position, however, and not all positions in the
           | philosophy of mathematics will agree with it.
           | 
           | This all might seem trivial or even nonsensical (as
           | philosophy of mathematics so often appears), but I merely
           | want to point out that the 'uncountability' of the real
           | numbers is not a consequence of the set of the natural
           | numbers being 'too small' to hold all the real numbers,
           | because they are 'large enough' to assign numbers to all
           | possible descriptions all real numbers that will ever be
           | expressed in language. Uncountability is a consequence of a
           | view that restricts Cantor's diagonalized number from the set
           | of the countable number _but still considers this
           | diagonalized number to be a real number_ (which again is only
           | unambiguously defined if it is not allowed to diagonalize
           | itself). There are however other possible philosophical
           | viewpoints which either include the diagonalized number in
           | the set of countable numbers (at the cost of including
           | ambiguous or paradoxical numbers) or reject the view that
           | Cantor 's diagonalized number should be considered to be a
           | real number in the first place.
           | 
           | tl;dr: Yeah, you can always name a real number for which a
           | particular counting rule does not work, but only as long as
           | there is agreement regarding the philosophical underpinnings.
           | Most mathematicians can probably be considered platonists and
           | from their standpoint the real numbers are obviously
           | uncountable, but that is by no means true for all positions
           | in the philosophy of mathematics.
        
             | pdonis wrote:
             | _> We can thus assign natural numbers to real numbers by
             | assigning numbers to their expressions in a natural or
             | formal language_
             | 
             | This doesn't work because not all real numbers _have_
             | expressions in a natural or formal language. This is easily
             | shown by an obvious variation on Cantor 's diagonal proof,
             | applied to your lexicographically ordered list of
             | expressions in any natural or formal language.
        
               | skoodge wrote:
               | Sure, that is why I wrote:
               | 
               | > In such a sense then, we can trivially "count" the real
               | numbers unless we hold the philosophical view that there
               | are real numbers that are not expressible. This is where
               | it becomes a question of philosophy of mathematics, not
               | mathematics proper.
               | 
               | I'm not disagreeing with your interpretation of Cantor's
               | diagonal proof, I'm merely pointing out that this
               | interpretation depends on a very specific philosophical
               | view of mathematics, namely the platonist view that the
               | real numbers _exist_ independently from their expressions
               | in any natural or formal language and that it makes sense
               | to say that there are real numbers that are not
               | expressible.
               | 
               | And yeah, nearly all working mathematicians will agree
               | with this view and from their perspective the real
               | numbers are uncountable, period, and you are right that
               | what I sketched "doesn't work".
               | 
               | But I think it's important to remember that there are or
               | could be alternative philosophical views of mathematics
               | that lead to a different interpretation, which will
               | reject not the mathematical validity of Cantor's diagonal
               | proof, but rather its _usefulness_ or _relevance_. After
               | all, how can you convince someone that there are real
               | numbers that are not expressible? By their very nature
               | they cannot be practically used in any calculation, so
               | how could you convince someone who is not convinced by
               | this philosophical assumption of Cantor 's diagonal
               | proof?
               | 
               | In other words, Cantor's diagonal proof cannot _prove_
               | that there are real numbers that are not expressible,
               | because the proof only makes (philosophical) sense if you
               | accept this viewpoint in the first place.
        
               | chmod775 wrote:
               | > It is obvious that all expressions in the English
               | language can be ordered from smallest to largest and
               | lexicographically, which makes these expressions
               | trivially countable.
               | 
               | You could say the same about just real numbers, which can
               | also be ordered from smallest to largest. This definitely
               | not 'trivially' implies countability.
               | 
               | Edit: I edited my post shortly after posting from "Humor
               | me and count out the first two real numbers".
        
               | skoodge wrote:
               | EDIT: The parent first said (before being edited, with my
               | original answer after the quote):
               | 
               | > Humor me and count out the first two real numbers in
               | English in lexicographical order.
               | 
               | Probably "one" and "six" (then "ten" and "two", followed
               | by all 4-letter expressions of real numbers), unless you
               | can think of an English expression with a length of three
               | letters or less that would occur before "one" and "six"
               | lexicographically. If you consider "pi" or "e" to be
               | descriptions of real numbers then clearly these would
               | occur before "one" and "six".
               | 
               | But of course you can also pick an arbitrary universal
               | Turing machine or other suitable formalism, pick an
               | encoding as bit strings and order expressions in such a
               | formal language according to their bit strings. Ordering
               | expressions in the English language is only the less
               | formal counterpart.
               | 
               | (If your point is that ordering expressions of real
               | numbers in English is far from unambiguous without first
               | agreeing on a dictionary of valid words and on rules of
               | what can be considered an expression of a real number in
               | more than one word then of course I would agree. But what
               | I'm driving at is not that it's easy to unambiguously
               | count out the real numbers in English, of course it's
               | not, but rather that any kind of natural or formal
               | language expressions, by being recursively enumerable,
               | are "numerous" enough to be a countable set of
               | expressions. To say that the real numbers are uncountable
               | is to accept the view that there are real numbers that
               | are not and can never be expressible in language, which
               | is a view that only makes sense against the backdrop of a
               | very specific philosophical framework. One that is
               | definitely accepted more or less implicitly by most
               | working mathematicians, but not the only possible one.
               | And this philosophical framework cannot itself be
               | justified or grounded by a mathematical argument such as
               | Cantor's diagonal proof.)
               | 
               | ---
               | 
               | Reply to the parent after edit:
               | 
               | > You could say the same about just real numbers, which
               | can also be ordered from smallest to largest. This
               | definitely not 'trivially' implies countability.
               | 
               | No, what I meant was that by ordering expressions in the
               | English language or other suitable formal languages first
               | from smallest string to largest string and within these
               | groups lexicographically, you can enumerate all the
               | expressions in such a language, which makes them
               | trivially countable.
               | 
               | Of course this does not give you a 1:1 mapping from
               | expressions in a language to real numbers, but it is
               | meant to illustrate that the real numbers are not
               | uncountable because they are 'too numerous' to be counted
               | by the natural numbers, in the sense that a box is not
               | large enough to hold a collection of things. They are
               | uncountable because we accept the view that there can be
               | real numbers that are not and cannot be expressible in
               | language, which is a platonist view that is open to
               | philosophical critique.
        
               | chmod775 wrote:
               | > Probably "one" and "six"
               | 
               | You missed infinitely many numbers between those two. For
               | example "one thousand" and "one dot/comma three".
               | 
               | Once you've worked that all out, can you now tell me
               | which natural number you assigned to "six" in your
               | lexicographical order?
               | 
               | Edit: Ah. Took me a moment to realize you are ordering by
               | number of characters first.
               | 
               | > They are uncountable because we accept the view that
               | there can be real numbers that are not and cannot be
               | expressible in language, which is a platonist view that
               | is open to philosophical critique.
               | 
               | So similar to R \ Q? Or the same?
        
               | skoodge wrote:
               | Oh, I see what the issue is. I wrote "smallest to largest
               | and lexicographically", meaning first from smallest to
               | largest (as measured by the length of the string) and
               | then within each group lexicographically, which is the
               | usual way of giving an enumeration of such expressions as
               | far as I know. Of course you cannot enumerate these
               | expressions if you expect a solely lexicographical
               | ordering. In any case, English was just an example, you
               | can basically pick any Turing-complete language that is
               | recursively enumerable and count its expressions by
               | considering its bit string encodings as natural numbers.
               | 
               | Edit: This is also why the computable numbers are
               | countable, but not computably enumerable (because
               | figuring out which expressions correspond to real numbers
               | is equivalent to the halting problem).
        
               | chmod775 wrote:
               | You've convinced me that R without all numbers that are
               | not expressible in typical languages is countable at
               | least.
               | 
               | Though I'm not convinced that numbers that are not
               | expressible don't exist. I could for instance say "the
               | length of this line", which may very well have a length
               | that is not expressible in a language that uses a finite
               | set of symbols (hah, that's why your expressions are
               | countable, of course!).
               | 
               | Consider a language however that instead of numbers
               | simply uses sounds of the appropriate length, or draws
               | lines of proportional length (assuming of course you
               | could do this precisely).
               | 
               | With that language you could express every real number,
               | and you could express numbers that turing machines* or
               | English can't.
               | 
               | *Unless programmed in that language.
        
               | skoodge wrote:
               | That's an interesting thought experiment, though I'm not
               | sure how using "sounds of the appropriate length" or
               | "lines of proportional length" would get you more than
               | the rational numbers, which are already countable and
               | thus fully captured by any Turing-complete language. To
               | say that there are inexpressible real numbers is to say
               | that there are numbers that are not rational but which
               | can never be practically used or accessed either, at
               | which point they become kind of like an invisible and
               | unnoticeable unicorn: it is certainly possible to believe
               | in its existence, but such a belief is quite different
               | from the belief in the existence of practically useful
               | real numbers such as pi.
               | 
               | I am not trying to convince anyone that inexpressible
               | real number do or do not exist, but I think it's worth
               | noting that these issues quickly cross over into the
               | realm of philosophy, where it's not possible to justify a
               | particular conviction by appealing to firm mathematical
               | or practical reasons. Nothing wrong with that, of course.
               | 
               | Personally, I'm content with what is expressible in
               | language and I consider mathematical concepts going
               | beyond this boundary of expressivity as inessential to my
               | own personal use, though I can certainly see that these
               | mathematical calculi can be of interest to mathematicians
               | on their own.
        
               | btilly wrote:
               | Your "easily shown" is only easily shown if you're
               | sloppy.
               | 
               | If you're careful, it can't be shown at all.
               | 
               | As I already pointed out, if classical mathematics is
               | consistent, then constructivism must be as well.
               | Therefore if you think that you've found a logical flaw
               | in constructivism, the mistake must be in your own
               | thinking.
        
         | klysm wrote:
         | Does constructivism rule out uncountability because induction
         | is always indexable?
        
           | btilly wrote:
           | There are different schools of constructivism. But if you
           | insist on only dealing with mathematical objects that admit
           | of finite descriptions with finite symbols, there is a
           | surjection from a countable set onto the entire universe of
           | possible mathematical objects.
           | 
           | That said, an _enumeration_ of all mathematical objects is
           | not possible. That 's because we may not be able to resolve
           | the question of whether 2 descriptions of a mathematical
           | object actually refer to the same object or not.
        
       | pavpanchekha wrote:
       | Great article. Since it looks like a lot of folks are interested
       | in this article, some extra background.
       | 
       | First, what is forcing? The article actually has a great
       | description of ultrapowers (a key part of the construction) but
       | it goes by a little fast, so you might like Tim Chow's "A
       | beginner's guide to forcing" [1] which does a good job not only
       | laying out the mathematical details at a high level, but also
       | really clearly explaining the philosophy of how you prove
       | something independent of the axioms. I found it very
       | illuminating.
       | 
       | Second, the article has some interesting notes on how
       | mathematicians go about what axioms to select and which not to.
       | Penelope Maddy's "Believing the Axioms" [2] is the classic on
       | this topic (it has two parts). It is focused on set theory, so it
       | has a nice description of Martin's axiom and the arguments for
       | and against. It was nice to read because it is a deeply technical
       | argument (set theory is hard!) but at the same time there is no
       | "right answer"--all of these axioms, after all, are independent
       | of ZFC, and it's "ok" to add any of them. The arguments are
       | sometimes aesthetic ("rules" versus "surprises"), sometimes
       | pragmatic (what they can and cannot prove), and sometimes involve
       | deep values of what the universe should be like (should higher
       | cardinalities be like lower ones? weirder? simpler?).
       | 
       | It might all seem abstract, but if your day job is programming,
       | imagine an argument over how to architect a large and complex
       | system. Perhaps both architectures are possible, but which one is
       | "right"? What arguments would you deploy? Set theorists are also
       | building a large and complex system (the universe of sets) and
       | are having arguments over how it should be built, which things it
       | should make easy and which hard, which technologies should be
       | supported natively (forcing?) and which should not.
       | 
       | [1] http://timothychow.net/forcing.pdf [2]
       | https://www.cs.umd.edu/~gasarch/BLOGPAPERS/belaxioms1.pdf
        
         | BoiledCabbage wrote:
         | Why does forcing work? To me it seems flawed (which obviously
         | means I don't understand it fully).
         | 
         | For diagonalization argument: 1) Assume every real can be
         | assigned a natural number. 2) Do a bunch of steps that
         | essentially find a new real that differs from any real you have
         | listed from step 1. 3) Conclude that either your steps are
         | flawed, or your initial assumption is wrong 4). Because your
         | steps aren't flawed then your initial assumption (that every
         | real can map to a natural number) is flawed.
         | 
         | That all makes sense to me. Forcing seems broken though: 1)
         | Capture of list/set of all real numbers 2) Do a bunch of steps
         | that essentially find a new real that differs from any real you
         | have listed from step 1. 3) Conclude that either your steps are
         | flawed, or your initial assumption is wrong 4). Because your
         | steps aren't flawed then your initial assumption (that you can
         | produce a list of every real number) is flawed.
         | 
         | Now you're left in an odd situation. Didn't you just conclude
         | that it's impossible to have a set of all real numbers? Which
         | isn't what you're trying to prove at all.
        
           | cwzwarich wrote:
           | In a forcing-based consistency proof of a proposition P, you
           | start with a model of ZFC (or a sufficiently large subset of
           | ZFC, depending on the exact formalism being used),
           | approximate a witness to P (e.g. a bijection between a set of
           | reals and aleph_2) within the starting model, and then apply
           | the forcing machinery to construct a new model of ZFC where P
           | holds. Unlike the diagonalization argument, it's inherently
           | metamathematical.
        
           | jerf wrote:
           | "Didn't you just conclude that it's impossible to have a set
           | of all real numbers?"
           | 
           | Cantor's diagonalization proof proves that it's impossible to
           | list all the real numbers with a list of size aleph-0, which
           | is the cardinality of the set of natural numbers.
           | 
           | The forcing proof is an attempt to prove you also can't do it
           | with the a list of the size aleph-1, which is the size of the
           | power set of aleph-0. It purports to prove that you need a
           | list of size aleph-2, which is the size of the powerset of
           | aleph-1.
           | 
           | You can kind of think of this not so much as whether "Can
           | Crysis run?" ("can you have a set of all real numbers?") but
           | "Can Crysis run _on this machine_? " Do the real numbers need
           | aleph-1 "resources", or aleph-2 "resources" to list?
        
             | glial wrote:
             | I think this is a typo:
             | 
             | > aleph-0, which is the cardinality of the set of real
             | numbers.
             | 
             | aleph-0 is the cardinality of the set of natural numbers,
             | and (as you say) is not the cardinality of the real
             | numbers.
        
               | jerf wrote:
               | Yes, thank you, that was a typo.
        
             | BoiledCabbage wrote:
             | Thanks for the reply. Question though. in Cantor's argument
             | we explicitly mapped the reals to aleph-0 so it makes sense
             | that our conclusion decides that mapping to aleph-0 is too
             | small so it's size must be larger.
             | 
             | Where in the forcing process do we even "use" aleph-1? If
             | we used aleph-1 then it could see the parallels and the
             | argument would make sense - but all I see in the forcing
             | process is "start with a set of all reals". Nothing about
             | trying to map them to aleph-1. Maybe it's implicit, but
             | couldn't I have just changed step 1 to instead be "start
             | with a list of size aleph-90, then use forcing to prove
             | that the reals are larger than aleph-90." In Cantor's
             | argument it feels like we "used" the natural numbers. Here
             | it seems more like we said take an arbitrary number of them
             | and see this one isn't in it. But that arbitrary number of
             | them (aleph-1) could've been any arbitrary number of them
             | because we never really used any property of the set being
             | aleph-1 vs aleph-90.
        
               | finnh wrote:
               | Great question, I have no answer.
               | 
               | The article explained forcing in such a way as to simply
               | restate what I thought we already knew: given a real,
               | there is no "next" real. (ie, there are a non-countable-
               | infinite number of reals between any two reals).
               | 
               | I don't see the newness that forcing brings to this.
        
               | [deleted]
        
               | dwohnitmok wrote:
               | Forcing is about a completely different question.
               | 
               | The question is not where there are reals between reals,
               | it's a question of the size of sets. In particular is
               | there a set strictly larger than the natural numbers, but
               | strictly smaller than the reals? Forcing allows us to
               | construct such a set in ZFC.
        
               | CamperBob2 wrote:
               | Not a mathematician, so this probably has an obvious
               | answer, but who says that "size" is a necessary universal
               | property of a set? It doesn't seem any more rational to
               | speak of size as a required property of a set than it
               | would be to treat color or weight that way. Some sets
               | have those attributes, but not all.
               | 
               | It seems perfectly reasonable to say that the set of real
               | numbers is one of those sets that doesn't have a "size"
               | property.
        
               | dwohnitmok wrote:
               | Size is a hand-wavy way of saying it. What we're really
               | asking is given two sets A and B, is there a function
               | from A to B such that no two elements from A are mapped
               | to a single element in B?
               | 
               | It turns out if you use ZFC (in particular choice) then
               | one direction must hold. Either such a function exists
               | from A to B or from B to A. This means that we can order
               | all sets using the existence of these functions, and so
               | in that sense size is a "universal property."
        
             | scotty79 wrote:
             | That's weird. I thought that it was proven that existence
             | of sets larger than aleph-0 but smaller than the number of
             | real numbers is undecidable and you can add it (or negation
             | of it) as additional axiom to math.
             | 
             | https://en.wikipedia.org/wiki/Cardinality_of_the_continuum
             | 
             | "The continuum hypothesis, which asserts that there are no
             | sets whose cardinality is strictly between aleph-0 and
             | c=aleph-1. The truth or falsity of this hypothesis is
             | undecidable and cannot be proven within the widely used ZFC
             | system of axioms."
        
               | mdoms wrote:
               | This is covered in the article.
        
             | Layke1123 wrote:
             | Cantor's diagonalization proof was never a proof to begin
             | with. You can't construct a number that isn't on an
             | infinite list of all numbers because you can't list all the
             | numbers, otherwise you'd eventually find your number. It's
             | honestly kind of ridiculous that this proof hasn't been
             | subjected to more rigor than just handwaving away the
             | problems.
        
             | avz wrote:
             | > aleph-1, which is the size of the power set of aleph-0
             | 
             | In ZFC, aleph-1 is not the size of the powerset of aleph-0.
             | Instead, aleph-1 is the next larger cardinal number after
             | aleph-0. The size of the powerset of aleph-0 is called
             | continuum or beth-1. In ZFC, we can show that the size of
             | the set of real numbers equals continuum, but it is not
             | possible to relate continuum to a specific aleph-k (though
             | some can be ruled out using Easton's theorem).
             | 
             | Now, the statement that aleph-1 is the size of the powerset
             | of aleph-0 is known as the Continuum Hypothesis (CH) and is
             | independent of the axioms of ZFC. Therefore, your claim
             | that aleph-1 is the size of the powerset of aleph-0 cannot
             | be made in ZFC. The statement can be made in ZFC+CH, but
             | then the question which aleph-k is the size of the real
             | numbers has a straightforward answer: aleph-1.
        
               | jerf wrote:
               | Thank you. I accept this.
        
           | myhf wrote:
           | > 2) Do a bunch of steps
           | 
           | It's not just a bunch of steps. It's an infinite number of
           | steps. It requires the axiom of choice.
        
           | HotHotLava wrote:
           | The illustration in the article doesn't start with the whole
           | set of all real numbers, but with just "a" set containing an
           | uncountable number of reals, so there is no contradiction in
           | that sense.
           | 
           | I'm assuming there's some unmentioned technical condition on
           | which sets you are allowed to choose in order for the
           | procedure to work, otherwise the argument would indeed seem
           | to lead to a contradiction when choosing the set of all real
           | numbers.
        
         | intuitionist wrote:
         | Maddy's work is great and worth reading (even as an
         | antirealist!), but it's worth keeping in mind that debates
         | about foundations are basically irrelevant to the working lives
         | of the large majority of mathematicians. If you're studying,
         | say, extremal graph theory or the Langlands program or low-
         | dimensional topology, the cardinality of the continuum is
         | simply not relevant.
        
           | pavpanchekha wrote:
           | Yes, I actually have a kind-of related blog post:
           | 
           | https://pavpanchekha.com/blog/proof-system-os.html
           | 
           | I tried to restrict my post to just set theory because this
           | philosophical point is often hard for people to grasp.
        
         | rssoconnor wrote:
         | > but at the same time there is no "right answer"--all of these
         | axioms, after all, are independent of ZFC, and it's "ok" to add
         | any of them.
         | 
         | Minor quible: Just because a potential axiom is independent of
         | ZF(C) doesn't make it necessarily "okay" to add. Potential
         | axioms can be unsound, for example if they prove new / untrue
         | Sigma_1 statements. As an example, even in the likely
         | circumstance that !Con(ZFC) is independent of ZFC, it wouldn't
         | be "okay" to add !Con(ZFC). While the resulting system would be
         | consistent, and does have models, the resulting system is
         | unsound (in the sense of Tarski) because it asserts the
         | existence of natural numbers that have no "written form" (i.e.
         | the existance of natural numbers that are larger than any term
         | you can write to denote a natural number).
         | 
         | That said, Martin's axiom, like the CH (or the axiom of
         | choice), does not have any arithmetic consequences, and thus
         | doesn't fall into this category of problematic axioms.
        
           | pavpanchekha wrote:
           | You're absolutely right. ZFC + !Con(ZFC) is a weird set of
           | axioms, and no mathematician really studies it. I skipped
           | this point, because we weren't really discussing such axioms,
           | but it's an important point that there are different levels
           | of mathematical / philosophical commitments, and realism and
           | PA are way stronger commitments than the ones people have
           | about cardinalities.
        
           | tgflynn wrote:
           | > he resulting system is unsound (in the sense of Tarski)
           | because it asserts the existence of natural numbers that have
           | no "written form" (i.e. the existance of natural numbers that
           | are larger than any term you can write to denote a natural
           | number).
           | 
           | Isn't that basically the definition of the natural numbers,
           | ie. if you write down any natural number (say n) I can always
           | construct a natural number that is larger than it (like n+1)
           | ?
        
             | kmill wrote:
             | This surprisingly doesn't mean repeatedly adding 1 will
             | exhaust all natural numbers -- there are models for the
             | natural numbers with elements that can't be reached this
             | way!
             | 
             | The ultrafilter construction gives one such model. You take
             | the set of all sequences of natural numbers (n1, n2, n3,
             | ...) then use an ultrafilter to decide which of these
             | sequences are considered to be equal. The usual operations
             | of natural numbers are defined term-by-term. You can think
             | of the usual natural numbers as being the sequences
             | (0,0,0,...), (1,1,1,...), (2,2,2,...) and so on. However,
             | there are many additional numbers in this system, like
             | (0,1,2,...) that are strictly greater than all the usual
             | numbers, and these cannot be reached by repeatedly adding
             | one. (The reason (0,1,2,...) is greater than (n,n,n,...) is
             | that if we do the comparison term-by-term we get
             | (false,false,...,false,true,true,true,...), and the
             | ultrafilter will decide the comparison is true because it
             | is true for all but finitely many terms. Ultrafilters are
             | devices to consistently turn infinite sequences of trues
             | and falses into a single decision, but every ultrafilter
             | will make a true decision in the case there are only
             | finitely many falses.)
             | 
             | Counter-intuitively, proofs by induction still work for
             | this system... speaking with no authority here, maybe an
             | intuition is that it's doing a hypercomputation.
        
               | logicchains wrote:
               | >The ultrafilter construction gives one such model.
               | 
               | It's worth noting that this construction relies on the
               | axiom of choice, so exists in ZFC but not ZF. Generally a
               | lot of counter-intuitive constructions disappear when
               | eliminating the axiom of choice (such as the Banach-
               | Tarski paradox and non-continuous functions).
        
               | kmill wrote:
               | > non-continuous functions
               | 
               | I think the step function doesn't depend on choice. Say
               | f(x) = 0 if x < 0 and f(x) = 1 if x >= 0. Also, suppose
               | we define the reals through Dedekind cuts. We can define
               | f(x) by checking whether 0 is an element of x as a cut.
               | We can prove f is discontinuous in the usual way.
               | 
               | What I had heard is that Brouwer proved, using some kind
               | of constructive/intuitionistic logic, that every function
               | is continuous, but ZF is still based in classical logic.
               | I believe the step function is not definable
               | constructively.
        
               | R0b0t1 wrote:
               | Can you explain what use these have? If natural numbers
               | are (n, n, n, ...) then you have just made a new type of
               | number not comparable to natural numbers.
        
               | kmill wrote:
               | A theoretical use is that it shows that the axioms of the
               | natural numbers (the Peano axioms) aren't enough to pin
               | down what we think the natural numbers should be -- there
               | are these crazy non-standard natural numbers out there!
               | 
               | In context of the discussion, this non-standard model of
               | the natural numbers gives some intuition about what
               | happens if the consistency of ZFC is independent of ZFC
               | and you add in the axiom that ZFC is inconsistent: your
               | natural numbers will have to be something similarly
               | weird.
               | 
               | > you have just made a new type of number not comparable
               | to natural numbers
               | 
               | But they _are_ comparable. The everyday natural numbers
               | are embedded in this new system, which is what these
               | (n,n,n,...) elements represent. One way to interpret what
               | we 've done here is to add in infinities to the number
               | system so that all the normal operations still work, and
               | there's even a total ordering on them!
               | 
               | Using the real numbers instead of the naturals, you get
               | the so-called nonstandard real numbers, which is the
               | number system used in nonstandard analysis. Nonstandard
               | analysis is a way to do analysis without epsilon-delta
               | proofs. Nonstandard reals include infinitesimals and
               | infinities, similar to nonstandard naturals having
               | infinities. There are even textbooks on the subject -- I
               | haven't read them, but they claim it makes analysis
               | easier.
               | 
               | The last thing that comes to mind is that nonstandard
               | numbers of this exact type show up when you study certain
               | infinite-dimensional number systems and calculate the
               | points (like in the end of my comment).
        
               | kmill wrote:
               | > calculate the points (like in the end of my comment)
               | 
               | Oh, I forgot I commented more than once today on HN. I
               | was referring to this one:
               | https://news.ycombinator.com/item?id=27851188
        
             | rssoconnor wrote:
             | Dropping down to Peano Arithmetic for a moment. We can
             | consider adding a new constant 'c' for a natural number to
             | the language along with the following infinite list of
             | axioms about this remarkable constant:
             | 
             | - 0 < c
             | 
             | - 1 < c
             | 
             | - 2 < c
             | 
             | - 3 < c
             | 
             | ...
             | 
             | Adding all these axioms is consistent. I.e. you can do
             | induction upto 'c', whatever it is. Why is it consistent?
             | Because if there was a contradiction, the proof of such a
             | contradiction would be finite, and hence can only use a
             | finite number of these new axioms (this is a so-call
             | compactness argument). But clearly any finite subset of
             | this list of axioms is consistent because it has a model
             | where c is just defined to be 1 more than the largest
             | numeral appearing in that list.
             | 
             | But all those infinite number of axioms taken together
             | creates an unsound system because it claims that 'c'
             | denotes a natural number that is larger than every written
             | numeral.
             | 
             | Heading back to ZFC land, it turns out that (assuming
             | !Con(ZFC) is independent of ZFC) adding !Con(ZFC) to ZFC
             | similarly is similarly unsound in that it yields only
             | models that have elements that are larger than every
             | written numeral.
        
               | skissane wrote:
               | > in that it yields only models that have elements that
               | are larger than every written numeral.
               | 
               | Are these like hyperintegers (or should I say
               | hypernaturals), as in the hyperrreal numbers used in non-
               | standard analysis?
        
             | hypersoar wrote:
             | You can find a natural number that is bigger than any one
             | natural number, but you can't write _one_ down that 's
             | bigger than _every_ natural number in the ZFC sense.
        
               | gooberwonder wrote:
               | 45,000,000,001?
        
               | dwohnitmok wrote:
               | > that's bigger than every natural number in the ZFC
               | sense.
               | 
               | I'm not sure what you mean by this.
               | 
               | Even for nonstandard models of the natural numbers there
               | will never be a single number larger than all other
               | numbers, since that violate the Peano axioms.
               | 
               | Did you mean by "in the ZFC sense" "the standard model?"
        
               | munk-a wrote:
               | Do you mean that given finite resources (like time and
               | matter) we would be unable to express the number? Or are
               | you talking about a sort of recursive thing where writing
               | down a number which is a sum of all natural numbers up to
               | and including itself is impossible since that number is
               | bigger each time you look at it?
        
             | l33t2328 wrote:
             | I don't believe a consequence of the axioms discussed is
             | that there exist natural numbers with no "written form." Do
             | you have a proof or citation?
        
             | chongli wrote:
             | If you can write down the natural number n, you can write
             | down n+1.
             | 
             | Of course, you can't write down the entire set of natural
             | numbers but _that_ is not a natural number.
        
               | skybrian wrote:
               | On any storage system you will eventually run out of
               | memory, so there will always be a maximum integer that
               | you can write down with what you have, and we can only
               | write down a vanishingly small subset of the natural
               | numbers.
               | 
               | Of course, this isn't what mean when they talk about
               | "writing down" a number, but I've long thought that
               | assuming the existence of natural numbers that you can't
               | write down is poetically apt.
               | 
               | In some sense, the set of natural numbers is "too big" to
               | be about everyday finite numbers alone and it seems
               | fitting to acknowledge that our usual axioms can't
               | exclude weirdo large finite numbers that are simply too
               | big to ever reach.
        
           | lupire wrote:
           | why is that "unsound"? what's wrong with an unwritable
           | natural? Almost all reals are unwritable.
        
             | rssoconnor wrote:
             | Mostly because it implies that some Turing machines halt
             | that actually do not halt. That is unless you are willing
             | to accept that a Turing machine can halt in some number of
             | steps that is beyond any number that can be written. And I
             | don't mean can't be written in the sense that we don't have
             | enough paper. Just cannot be written in principle at all by
             | our notation for numbers.
        
               | bryan0 wrote:
               | But isn't this what the busy beaver numbers are? Numbers
               | that we cannot write for arbitrary n but they do exist?
        
               | thaumasiotes wrote:
               | Do they exist? I thought there was an independence
               | theorem for the specific values of almost all busy beaver
               | numbers.
        
               | bryan0 wrote:
               | Yes they are independent of ZFC but they still exist.
               | From the original article talking about CH which is also
               | independent:
               | 
               | > This independence is sometimes interpreted to mean that
               | these questions have no answer, but most set theorists
               | see that as a profound misconception. They believe the
               | continuum has a precise size; we just need new tools of
               | logic to figure out what that is. These tools will come
               | in the form of new axioms.
               | 
               | I believe the same applies for BBs. They have precise
               | values outside of ZFC but we still cannot ever know them
               | because they are incomputable (??)
        
               | thaumasiotes wrote:
               | > but we still cannot ever know them because they are
               | incomputable (??)
               | 
               | That doesn't mean you can't know them. We know BB(2). It
               | means there is no single algorithm which is capable of
               | yielding them all. But there could, theoretically, be an
               | algorithm for BB(10), a different algorithm for BB(11),
               | etc.
               | 
               | In fact, those individualized algorithms don't exist
               | either.
        
               | rssoconnor wrote:
               | They exist in that it is easy to prove "for ALL x there
               | EXISTS y such that (there EXISTS a Turing machine with at
               | most x states M such that (there EXISTS some n such that
               | M prints y 1's and halts after n steps) AND (for ALL
               | Turing machines with at most x states M, if (there EXISTS
               | some n such that M halts after n steps) then (M prints no
               | more than y 1's after n steps))". I expect you can prove
               | this in systems as weak as Primitive Recursive
               | Arithemetic.
               | 
               | It is true though that various (decidable) proof systems
               | are unable to prove that the Busy Beaver function has any
               | specific value beyond a certain point. However, where
               | that cut-off is varies from proof system to proof system,
               | with stronger proof systems being able to prove more an
               | more values of the Busy Beaver function.
               | 
               | Maybe you find it weird that we can prove (Exists x. P x)
               | without being able to prove P n for any particular
               | numeral n? Welcome to the strange world of classical
               | (i.e. non-constructive) mathematics.
        
               | thaumasiotes wrote:
               | > It is true though that various (decidable) proof
               | systems are unable to prove that the Busy Beaver function
               | has any specific value beyond a certain point.
               | 
               | Isn't the result stronger than that though? The value of
               | BB(30) might be X. Or it might be Y. Neither value would
               | cause any problems.
               | 
               | Thus, "the value" doesn't exist. There is no value that
               | is the value of BB(30).
        
               | rssoconnor wrote:
               | I think this is a great question.
               | 
               | The difference here is that with busy beaver numbers,
               | e.g. BB(101) we can, presumably, write their values with
               | our notation; it's just that we often cannot prove that
               | any particular value written in our notation does indeed
               | denote the value for that function. So if we write
               | 100000...0000 with an unholy number of 0s there, it might
               | be the value of BB(101), in particular we might not be
               | able to prove that it isn't.
               | 
               | On the other hand, for a non-standard number, c, it is
               | definitely the case that 100000...0000 is not c, because,
               | whatever c is, it is strictly greater than 100000...0000,
               | or any other number we can write down.
               | 
               | And thus when it comes to proofs about the termination of
               | Turing machines that do not actually terminate, the
               | unsound system is claiming that some machine terminates,
               | but it doesn't terminate in 1 step, nor 2 steps, nor 3
               | steps, nor ... nor 100000...0000 steps, nor 100000...0001
               | steps, nor 100000...0002 steps, nor .... However,
               | regarding the BB(101), the (presumably) sound systems we
               | use such as PA, or ZFC, do not claim that BB(101) isn't
               | 100000...0000. They just may not be able to prove
               | anything one way or the other.
        
             | dogecoinbase wrote:
             | Soundness in this sense (sigma_1 soundness) is defined by
             | equivalency to the standard model (specifically, all
             | sentences provable in the system must be provable in the
             | standard model).
        
               | dwohnitmok wrote:
               | It is defined by equivalency to _a_ standard model.
               | Whether there is a single standard model is a
               | philosophical question (which granted most, but not all,
               | set theorists tend to agree with). Hence sigma_1
               | soundness is from a purely mathematical point of view a
               | relative statement.
        
             | pavpanchekha wrote:
             | I wonder what you mean by "what's wrong with an unwritable
             | natural"... It is sound to assume one! And it doesn't make
             | any true mathematical facts false. But of course it's sound
             | to assume many silly things. But why assume those things?
             | 
             | The reality is that mathematicians didn't first come up
             | with the Peano axioms and then study the interesting
             | consequences of them. Both as a matter of history and also
             | as a matter of why most mathematicians do math,
             | mathematicians first came up with numbers, and only later
             | came up with axioms that allow them to do rigorous
             | reasoning about them. The same is actually true of almost
             | all math--real numbers were invented to do rigorous
             | reasoning about calculus, which by that point had been
             | around for a century plus; set theory likewise.
             | 
             | In other words, if the goal is to come up with any old
             | consistent set of axioms, you can assume an unwritable
             | natural number, that's fine, go ahead. But if your goal is
             | to study natural numbers--which, like, I have a lot of
             | experience with natural numbers in my day to day life, I'm
             | pretty sure I could write all of them if I had enough time
             | and space and so on--then you want to study natural
             | numbers, not some other weird things where there are
             | unwritable weird things.
        
             | dwohnitmok wrote:
             | This is not unsound in the usual sense of "logical
             | soundness" (which applies to logical systems such as first-
             | order logic, rather than specific theories in the system
             | such as ZFC).
             | 
             | This is unsound in that it runs counter to certain
             | philosophical commitments, namely that there should be some
             | tangible, physical realization of all the natural numbers
             | (although if you really go far in that direction you end up
             | with ultrafinitism, which most set theorists would find
             | unpalatable, so the philosophical implications of all this
             | are rather tricky).
        
           | Someone wrote:
           | > _because it asserts the existence of natural numbers that
           | have no "written form"_
           | 
           | I don't see why that should imply it wouldn't be "okay" to
           | add !Con(ZFC).
           | 
           | It may be highly counterintuitive, but the history of
           | mathematics is full of counterintuitive results that nowadays
           | are accepted as true in mainstream mathematics.
           | 
           | Well-known examples are the existence of irrational numbers,
           | the claim that the set of natural numbers has the same size
           | as that of the rational numbers, the existence of hyperbolic
           | geometry, and the Banach-Tarski paradox.
        
             | rssoconnor wrote:
             | It's not "okay" because such a system proves that various
             | particular Turing machines halt when, in fact, those
             | machines do not halt. See
             | https://news.ycombinator.com/item?id=27847719.
             | 
             | But to be a bit more specific, !Con(ZFC) says that the
             | Turing machine that searches for a contradiction in ZFC
             | does indeed halt. However (in all likelihood) such a
             | machine does not actually halt, in the sense that it does
             | not halt in 1 step, and it does not halt in 2 steps, and it
             | does not halt in 3 steps, etc., and indeed (in all
             | likelihood) for each numeral n, ZFC even proves that the
             | machine does not halt in n steps.
             | 
             | (Now there is a small possibility that maybe such a machine
             | does halt in some particular number of steps. If it does
             | actually halt, that means it has found a proof that ZFC is
             | inconsistent. But this scenario is even worse, because it
             | means that ZFC itself is not only unsound, but
             | inconsistent, (and hence ZFC+!Con(ZFC) is also
             | inconsistent). In particular ZFC+!Con(ZFC) being
             | inconsistent means it proves that every Turing machine
             | halts, which is even more wrong in general, even if it
             | happens to be right about this particular machine.)
        
               | a1369209993 wrote:
               | > such a system proves that various particular Turing
               | machines halt when, [ _]in fact[_ ], those machines do
               | not halt.
               | 
               | Er, no. The _fact_ is that there _is no_ fact of the
               | matter as to whether those particular Turing machines
               | either a: do not halt at all, or b: halt after a
               | (colloquially) infinite number of steps. (A implication
               | of there being no fact of the matter is that,
               | empirically, we can 't tell the difference by running
               | them, but we can't tell the difference for a machine that
               | halts in 2^(2^(2^256)) steps (or not at all) either, so
               | that's not very interesting on it's own.)
               | 
               | (As you note, we don't actually _know_ that (for example)
               | a Turing machines looking for contradictions in ZFC is
               | one of those particular machines; indeed I 'm not sure
               | offhand if we actually know of _any_ specific example of
               | such a machine. But that 's presumably not the issue
               | here.)
        
               | rssoconnor wrote:
               | ZFC+!Con(ZFC) either wrongly proves that the machine that
               | searches for an inconsistency in ZFC halts, or it wrongly
               | proves that `while(true)` halts. Either way ZFC+!Con(ZFC)
               | is wrong about something.
        
             | graycat wrote:
             | The Banach-Tarski paradox is fine with me: Not all subsets
             | of the real line or R^n for the set of real numbers R and
             | positive integer n are measurable. Yes, the usual example
             | of a non-measurable set uses the axiom of choice. The sets
             | in the Banach-Tarski paradox are not measurable -- okay.
             | 
             | Of course the natural numbers can be put into 1-1
             | correspondence with the rationals -- how to show that is
             | the classic Cantor diagonal argument.
             | 
             | At a tea before a seminar, I asked Max Zorn what Paul Cohen
             | had just proved. Zorn didn't explain it and instead just
             | loaned me his copy of Cohen's paper -- I lost it in a move!
             | If Zorn wasn't strongly interested in Cohen's proof, then
             | neither should I be.
             | 
             | For any set of axioms, there will be statements we can
             | neither prove nor disprove. Surprising, interesting, got
             | it.
             | 
             | Zermelo-Fraenkel set theory with the axiom of choice seem
             | fine to me -- now back to the real work!
        
             | hutzlibu wrote:
             | "the claim that the set of natural numbers has the same
             | size as that of the rational numbers"
             | 
             | Where can I read more about that? Because, both are
             | infinite, but there still should be more rational numbers,
             | than natural numbers?
        
               | jari_mustonen wrote:
               | You can count rational numbers and everything countable
               | is the size of infinity as natural numbers.
               | 
               | Google "counting rational numbers" and "different sizes
               | of infinity" to learn more.
        
               | [deleted]
        
               | TheCondor wrote:
               | The proof is when you can create a bijective function
               | between the sets then they have the same cardinality. The
               | "zip" function can do this between integers and
               | rationals.
        
               | quisnam wrote:
               | Many entry-level real analysis courses will cover
               | cardinality after introducing sets and functions. There's
               | also a short article on Wikipedia. [0]
               | 
               | Your intuition about there being more rational numbers
               | might be based on viewing the rationals as a proper
               | superset of the natural numbers. You might similarly
               | consider that there are more natural numbers than even
               | natural numbers. However, by "renaming" every even
               | number, and that shouldn't change how many there are, to
               | half its value, we obtain the natural numbers. Formally,
               | there exists a bijection between N and 2N, as between N
               | and Q, and this is what mathematicians mean when they say
               | that sets have the same size, or cardinality.
               | 
               | [0]: https://en.wikipedia.org/wiki/Cardinality
        
               | Someone wrote:
               | To add, for hutzlibu and others like them: as soon as
               | infinite sets (or sequences. See
               | https://plus.maths.org/content/when-things-get-weird-
               | infinit...) are involved math gets counter-intuitive.
               | 
               | In this case, at most one of these two a priori quite
               | reasonable statements can be true:
               | 
               | - if set B is a strict superset of A, B is larger than A.
               | 
               | - if you can map the times in A to the items in B in 1:1
               | fashion, A and B have the same size.
               | 
               | Giving up the first is deemed less problematic than
               | giving up the second (probably because that means giving
               | up comparing sizes of infinite sets at all, but I'm not
               | familiar enough with that to be sure about that)
        
           | thaumasiotes wrote:
           | > the resulting system is unsound (in the sense of Tarski)
           | because it asserts the existence of natural numbers that have
           | no "written form" (i.e. the existance of natural numbers that
           | are larger than any term you can write to denote a natural
           | number).
           | 
           | Isn't this a straightforward implication of nonstandard
           | analysis?
        
         | scrubs wrote:
         | @pavpanchekha: thank you indeed for this great post and great
         | references. I took ML and ran into CH, Ultrafilters but never
         | really got my head around it. Reading with interest!
        
         | 01GOD wrote:
         | Hmmm...somebody named "chow" faking something...why is that not
         | surprising?
        
         | pdonis wrote:
         | Thanks for these references! The Chow paper is the first one
         | I've read that makes me feel like I understand forcing and
         | Cohen's proof.
        
       | dwohnitmok wrote:
       | Man there's a lot of juicy stuff in this article (Woodin's
       | Ultimate L program gets briefly alluded to at the end of the
       | article).
       | 
       | I just want to point out, because the HN crowd seems to generally
       | not be mathematical Platonists, that this entire article is
       | implicitly assuming a Platonist philosophical foundation. This
       | may cause confusion for lay readers who are not mathematical
       | Platonists.
       | 
       | In other words the article assumes that mathematical objects have
       | an objective existence: they either exist or they do not. Hence
       | every single logical axiom has a truth value. You do not get to
       | arbitrarily choose what logical axioms you want. If you do, you
       | can end up choosing the "wrong one." Therefore it is an important
       | question to understand whether the Continuum Hypothesis is true
       | or not, even if it's independent of ZFC (and hence requires
       | ultimately philosophical rather than mathematical arguments).
       | 
       | If you aren't a Platonist and instead view logical axioms as
       | having no inherent truth value, but rather foundations that you
       | can pick and choose from as necessary (where in one case you may
       | choose to use an axiom and in another you may choose its
       | negation), then all that might sound very strange to you. In that
       | case, you should mentally substitute every instance of "true" or
       | "false" in the article with "agrees or disagrees with the meta
       | model used to examine the semantics of a logical theory." In
       | particular, whenever we talk about a formal treatment of the
       | semantics (i.e. model) of a theory, whether that be something
       | like ZFC or a programming language, we must always make those
       | statements relative to a meta-model.
       | 
       | For example if we talk about the formal semantics of a language
       | like C, we must first posit a meta-model which already contains
       | notions of things like "integer" and "natural number" which can
       | be used to give meaning to statements such as "performed an
       | operation n times." If you're not a Platonist then you probably
       | believe that there are multiple possible meta-models you could
       | use.
        
         | red_trumpet wrote:
         | Thanks for that perspective! Searching around I found this
         | article https://plato.stanford.edu/entries/platonism-
         | mathematics/#Tr... which even distinguishes mathematical
         | Platonism and truth-value realism. Interesting stuff!
        
           | dwohnitmok wrote:
           | Indeed, that article is correct. More accurately I should be
           | calling this position mathematical realism (although the
           | lines between the two can get a bit blurry).
        
         | IngoBlechschmid wrote:
         | Indeed! And in fact, there are set theorists who argue that the
         | traditional dream solution to the continuum hypothesis,
         | adopting new axioms, is doomed to fail, and that we should
         | instead embrace all models of set theory, all possible
         | universes of mathematics.
         | 
         | I tried to write an introduction to this "set-theoretic
         | multiverse philosophy" which is accessible to the HN crowd
         | here:
         | 
         | https://iblech.gitlab.io/bb/multiverse.html
        
         | kachnuv_ocasek wrote:
         | Thanks for the clarification. I was indeed a bit confused by
         | some of the implicit assumptions as an intuitionist myself.
        
         | goldenkey wrote:
         | Individually, or independently, axioms have no truth value. But
         | when you put together a system of multiple axioms, they can
         | contradict eachother. I see no problem with the question of
         | whether the continuum hypothesis is true, given ZFC as a
         | precondition. All we are asking is if the axiom contradicts
         | ZFC. Axiom independence is very similar to operator commutation
         | in quantum mechanics.
        
           | dwohnitmok wrote:
           | The Continuum Hypothesis and its negation are both proven to
           | be consistent with ZFC (this is what the article is talking
           | about RE forcing and Godel's proof of the consistency of CH).
           | There is no contradiction to assume one or the other
           | alongside ZFC. The article is really talking about Platonic
           | truth when it talks about something being true or false.
           | 
           | > Individually, or independently, axioms have no truth value.
           | 
           | Indeed, you are not a Platonist :).
        
             | IshKebab wrote:
             | Isn't that what an axiom _is_ though? Something you have to
             | define as true? The way you 've described it, Platonism
             | makes no sense. Maybe there's a missing part of the
             | explanation...
        
               | bonoboTP wrote:
               | Platonists think you can define them wrong if you're
               | careless.
               | 
               | Imagine having a spreadsheet with the population of each
               | country. I can go and enter 1 billion for the US, and the
               | spreadsheet will happily recalculate all the cells about,
               | say, GDP per capita etc. based on my incorrectly modified
               | input. Clearly just because I've set the US population
               | value in my spreadsheet to 1 billion the actual
               | population did not change. There _is_ a real population
               | count and if we don 't use the correct one then all our
               | output will be garbage too: garbage in, garbage out.
               | 
               | Mathematics is like this spreadsheet, its input are the
               | axioms and the outputs are various interesting theorems.
               | Feed in wrong axioms and you get wrong theorems.
               | 
               | Platonists believe in a realm of ideas, that math talks
               | about something real, just like the spreadsheet talks
               | about real human populations. Even though within the
               | spreadsheet everything remains fine and consistent if I
               | tweak the US population. It just doesn't correspond any
               | more to reality.
               | 
               | I myself am not a mathematical Platonist, I more of an
               | engineer/practical person and see math as a survival tool
               | of a bunch of primates on a rock floating in space, not
               | something that taps into anything mystical.
               | 
               | But the Platonist worldview also seems coherent.
        
               | dwohnitmok wrote:
               | bonoboTP gives a nice high-level description, I give a
               | slightly more ranty description here:
               | https://news.ycombinator.com/item?id=27857728.
               | 
               | Basically it seems like no matter what axioms you have,
               | there's a bright line as to whether they accurately
               | describe the natural numbers as we observe them in our
               | universe, as that has physical ramifications. That bright
               | line seems like "whether these axioms are true in our
               | universe" and that seems to lend an objective standard by
               | which we can measure our axioms.
        
         | zajio1am wrote:
         | > this entire article is implicitly assuming a Platonist
         | philosophical foundation
         | 
         | Is mathematical platonism still significant position between
         | mathematicians? I thought it is outdated since Lobachevsky.
        
           | dwohnitmok wrote:
           | It is a significant position among mathematicans and
           | logicians who study the foundations of mathematics.
           | 
           | It is probably not how most non-foundations mathematicians
           | think of mathematics (especially larger infinities).
        
           | zenon wrote:
           | In this survey, most "Philosophers of mathematics" endorsed
           | Platonism: https://philpapers.org/surveys/results.pl?affil=Ta
           | rget+facul...
        
             | saithound wrote:
             | edit: I did not pay enough attention, so what I wrote is
             | wrong, see the comment below
             | 
             | In the survey you linked, it looks like 45.7% of the
             | surveyed philosophers of mathematics endorse Platonism.
             | That's a lot, but not "most", by any measure.
        
               | oehtXRwMkIs wrote:
               | You're looking at the result for aesthetic value not
               | Platonism.
               | 
               | And by the measure of "most" meaning majority it would
               | still be most.
        
           | saithound wrote:
           | Note that a view can be a majority position even if it is
           | extremely outdated. This often happens when the outdated
           | position is much easier to explain than the more nuanced
           | alternatives. E.g. when surveyed, evangelical preachers often
           | endorse Arianism, despite it being considered outdated since
           | 325AD.
        
             | dwohnitmok wrote:
             | Even though I'm personally not much of a realist, I feel
             | like it's worth explaining why it's so popular when
             | studying foundations of mathematics, because outdated is
             | selling it short and it's actually a fairly nuanced
             | position. The basic question is are mathematicians
             | fundamentally discovering mathematics (Platonism) or
             | creating mathematics (non-Platonism)?
             | 
             | The first step is that it seems like natural numbers are
             | "real" and "tangible" in some sense. So for example, even
             | though we can have modular arithmetic that would say maybe
             | 2 + 5 = 2 under addition mod 5, it clearly feels
             | "artificial" compared to 2 + 5 = 7. More concretely there
             | seems to be some universal idea that seems to underly the
             | fact that if you have three stones and add another stone
             | you get four stones, or if you have three bottles and add
             | another bottle you get four bottles. "three", "one", and
             | "four" might not exist in the same as "bottle" or "stone"
             | exist, but clearly they seem to exist and are "true" in
             | some sense (it is clearly incorrect to say if you have
             | three bottles and add another bottle you still have three
             | bottles).
             | 
             | Indeed, to even talk about mathematics it seems like you
             | need some intuitive grasp of natural numbers that precedes
             | mathematical axioms, or at least be able to understand at a
             | deep level that if you have n things and you add another
             | thing, then you have n + 1 things, and that any axioms that
             | violate this are more "artificial" than axioms which don't
             | violate this. More generally, if you have axioms that
             | create results which violate the rules of "normal" natural
             | numbers, it seems reasonable to say that those axioms are
             | "artificial" and not "real" in some sense. And so one
             | interpretation of mathematicians' jobs, at least when
             | tightly scoped to natural numbers, is to find those axioms
             | of the natural numbers that reflect our natural numbers
             | really work in "our universe" as opposed to a hypothetical
             | other universe. And that approach seems to presuppose that
             | natural numbers exist, at least in our universe, in some
             | objective some that we are "discovering" with our axioms,
             | rather than "creating" ex nihilo.
             | 
             | Indeed being able to say the term "normal natural numbers"
             | and have even lay, non-mathematicians understand what that
             | means (as opposed to "weird natural numbers" such as
             | modular arithmetic) seems to suggest a certain
             | objectiveness to the natural numbers.
             | 
             | Or to put it another way, the abstract notion of "counting"
             | seems to exist objectively and outside of our formal
             | mathematics theories, and our formal theory of natural
             | number is really trying to describe "counting" rather than
             | create the notion of "counting" from scratch.
             | 
             | Now if you accept that "counting" as an abstract idea
             | exists in some sense that makes it possible to say either
             | "yes that is an accurate description of counting" or "no
             | that is not an accurate description of counting," then
             | Godel's incompleteness theorems become quite interesting,
             | namely that every useful theory of the natural numbers will
             | have additional axioms that are independent of the ones
             | already included in that theory. The non-realist looks at
             | the incompleteness theorems and says, "Well yeah those new
             | axioms just create different 'natural numbers' no biggie."
             | The realist says, "That seems totally at odds with the fact
             | that even though hypothetically you could have multiple
             | notions of 'natural number,' there is only one notion of
             | 'natural number' that holds true in our current universe
             | and agrees with our similarly abstract notion of 'counting'
             | and not a hypothetical other universe!"
             | 
             | And if you accept that for the natural numbers... well a
             | lot of things have ramifications for the natural numbers.
             | One classic example, as mentioned elsewhere in this HN
             | discussion, is the busy beaver function, a function whose
             | input is the number of instructions in a given Turing
             | machine and whose output is the maximum number of steps the
             | machine could possibly take for any input before it must be
             | non-terminating on that input (basically an end run around
             | the halting problem).
             | 
             | Any set of axioms (including ZFC) that includes the notion
             | of arithmetic makes a statement on what it think a finite
             | number of busy beaver values are (and only a finite number,
             | again the halting problem prevents us from knowing more).
             | And yet it seems like _every_ value of the busy beaver
             | function has some objective truth value in our universe! I
             | can just run the Turing machine and find out! And that
             | seems like an objective yardstick that we can use to
             | determine whether a given axiom system is  "true" or not in
             | our universe (granted it's not one of very much utility
             | because you are waiting to see if a machine runs forever,
             | but still something that seems to have obvious physical
             | ramifications).
        
               | zajio1am wrote:
               | > The basic question is are mathematicians fundamentally
               | discovering mathematics (Platonism) or creating
               | mathematics (non-Platonism)?
               | 
               | My main issue with mathematical platonics is not whether
               | we are discovering or creating mathematic (which seems to
               | me as semantically empty question), but whether existence
               | of matematical object is an absolute property, or a
               | property relative to a specific model/structure, and
               | whether some models are metaphysically exceptional, or
               | all models are metaphusically equal, only some are more
               | convenient to use, so they are more worth studying.
               | 
               | Consider some poly-platonist, who thinks that both
               | natural-number-structure and non-standard-number
               | structure (and both models of ZFC+CH and models of
               | ZFC+non-CH) exist and are "real" and "tangible", and we
               | discover internal relations in these structures as
               | mathematical knowledge.
               | 
               | > Or to put it another way, the abstract notion of
               | "counting" seems to exist objectively and outside of our
               | formal mathematics theories, and our formal theory of
               | natural number is really trying to describe "counting"
               | rather than create the notion of "counting" from scratch.
               | 
               | I can say that we all have clear informal concept of what
               | are natural numbers, just limitations of logic prevent us
               | to formalize that.
               | 
               | But i cannot see how such argument can be extended to set
               | theory, where are plenty of arbitrary choices how to
               | axiomatize that (e.g. ZFC vs. New Foundations).
        
               | dwohnitmok wrote:
               | To keep my Platonist hat on... Well if you accept that
               | there is an objective truth to the natural numbers, the
               | set theory axioms you choose will have ramifications for
               | the natural numbers. (Elsewhere in these threads a
               | commentator brings up that Not(Con(ZFC)) will tell you a
               | Turing machine terminates when in our universe it
               | doesn't)
               | 
               | Put another way there will always be a purely
               | arithmetical statement of the natural numbers that is
               | independent of your set theory axioms. What is the truth
               | value of that statement? If you think you have a clear
               | and absolute conception of the natural numbers it seems
               | like you should believe that an arithmetical statement
               | has an objective truth value, and if that's true, then
               | that seems like your objective benchmark to decide on how
               | to accept a new axiom (does it prove or refute this true
               | arithmetical statement?). There's your absoluteness.
        
           | a1369209993 wrote:
           | I thought it was outdated since Epimenides. It requires the
           | axiom of the excluded middle, which is about as false as
           | axioms can possibly be, on account of having concrete
           | counterexamples. (eg "This proposition is false.")
        
             | drdeca wrote:
             | Intuitionistic logic, which is what you get from not
             | assuming lem, still reaches a contradiction if you permit
             | that as a valid proposition. So, doing away with LEM isn't
             | sufficient. You could use a logic that explicitly has more
             | values though.
        
       | kkylin wrote:
       | Those with a bit more math background may enjoy Cohen's own
       | perspective on the discovery of forcing:
       | https://projecteuclid.org/journals/rocky-mountain-journal-of...
       | 
       | (Not technical as these things go, but helps to have a little
       | knowledge of logic / set theory and some amount of "mathematical
       | maturity.")
        
       | zeven7 wrote:
       | What online class should I take to learn more about this topic in
       | particular?
        
       | phkahler wrote:
       | You can map all the real numbers to the interval 0 <= x < 1. To
       | map one of those reals to an integer, simply write it's trailing
       | digits in reverse order on the left side of the decimal point.
       | You may then drop any leading zeros.
       | 
       | 0.0 => 0
       | 
       | 0.1 => 1
       | 
       | 0.2 => 2
       | 
       | ...
       | 
       | 0.14159 => 95141
       | 
       | The argument against this is that there are real numbers with an
       | infinite number of digits, which will not have a specific integer
       | associated with them. Or do they? Can there be an "infinitely
       | large integer?" or are there just infinitely many of them? I
       | think this question gets to the core of the problem.
        
         | panic wrote:
         | _> Can there be an  "infinitely large integer?"_
         | 
         | Check out _p_ -adic numbers (https://math.uchicago.edu/~tghyde/
         | Hyde%20--%20Introduction%2...), which are related to the
         | two's-complement representation of signed integers. For
         | example, in the 10-adic numbers, ...9999 + 1 = 0.
        
         | plus wrote:
         | > The argument against this is that there are real numbers with
         | an infinite number of digits, which will not have a specific
         | integer associated with them. Or do they?
         | 
         | If it was possible to construct your mapping, then there would
         | be a well-defined sorting of the reals between 0 and 1 based on
         | their integer representation (e.g. we could sort the set {0.05,
         | 0.1, 0.2} => {50, 1, 2} to [0.1, 0.2, 0.05] => [1, 2, 50]).
         | 
         | How would you sort the list {0.5, 1 / sqrt(2), pi - 3}?
        
           | [deleted]
        
           | DangitBobby wrote:
           | > then there would be a well-defined sorting of the reals
           | between 0 and 1 based on their integer representation
           | 
           | Why is that?
        
             | alanbernstein wrote:
             | Because the integers are ordered?
        
               | DangitBobby wrote:
               | I took well-defined to mean "if it exists, we know what
               | it is." So really I guess what I want to know is why it
               | matters that we can't actually calculate the mapping.
        
         | question000 wrote:
         | "Can there be an infinitely large integer"
         | 
         | No there can't there's always a larger integer. Anyone who
         | claims otherwise is just making stuff up. It's just mind
         | blowing to me that anyone think otherwise, it's like asking
         | what the final digit of pi is, you'd get laughed out of any
         | college class for insisting there might be one, rightly so.
        
         | red_trumpet wrote:
         | > You can map all the real numbers to the interval 0 <= x < 1.
         | 
         | Even easier use a well-established function such as arctan to
         | map the reals bijectively to the interval -p/2 < x < p/2, and
         | then scale and shift the interval to 0 < x < 1.
        
           | plus wrote:
           | This allows you to map the real range (0, 1) to (-infinity,
           | infinity). It does not allow you to map the integers to the
           | reals.
        
         | victorbojica wrote:
         | But if you assign them randomly, then it should work
         | 
         | 1->5.85916
         | 
         | 2->8.7599
         | 
         | ...
         | 
         | Or even randomize the order for the integers
         | 
         | 5->7.52256951
         | 
         | 77->848.455
         | 
         | ...
        
           | bonzini wrote:
           | You can't, this is Cantor's diagonal counterexample.
        
         | perfunctory wrote:
         | > simply write it's trailing digits in reverse order
         | 
         | so how do you map say pi/4? Which digit do you start with?
        
           | phkahler wrote:
           | That would be:
           | 
           | ...61893587
           | 
           | We start writing the digits right to left an pi/4 is about
           | 0.78539816...
        
             | perfunctory wrote:
             | So you found a flaw in Cantor's diagonal argument?
             | 
             | https://en.wikipedia.org/wiki/Cantor%27s_diagonal_argument
        
               | drdeca wrote:
               | They are just saying "but what if we replace the set of
               | integers with a larger set". I think they are also
               | defining this larger set in a base 10 specific way.
               | 
               | I think they basically mean the 10-adic numbers, which,
               | unlike the p-adics for p a prime number, uh, I forget
               | exactly what goes wrong, but I suspect it has like, zero
               | divisors or something like that.
        
         | gowld wrote:
         | > Can there be an "infinitely large integer?"
         | 
         | What could that even mean? Aren't integers all defined as
         | Succ^n(x) for a finite n and a base x?
        
         | pwdisswordfish8 wrote:
         | What does  1/3  map to?
        
       | pelorat wrote:
       | The existence of addition implies that the answer is infinity.
       | Too simple an explanation for mathematicians obviously.
        
         | Sharlin wrote:
         | The whole point of the article is that we don't know _which_
         | infinity it is.
        
         | howderek wrote:
         | They know it is infinite. They want to understand the
         | cardinality. Any two sets (including infinite sets) A, B have
         | the same cardinality if there exists a bijection from A|->B
        
         | throwawayboise wrote:
         | I didn't read the paper, and have only an undergraduate math
         | background, but it seems to me that for sufficiently large n, n
         | + 1 = n. Vague justifications related to limits.
        
       | 123pie123 wrote:
       | I've always been brought up to realise that math(s) is not real.
       | It's more of a (extremely) good system to model stuff, say like a
       | map isn't real, but just a representation
       | 
       | so this proof is pushing the system of this math(s) system
        
         | svachalek wrote:
         | The deeper physics goes, the more apparent it is that our
         | perception of reality is a good model for what is happening at
         | our scale in the universe and the forces that are dominant at
         | that scale. Our intuition breaks down at larger and smaller
         | scales, but the math still works. So in some sense, math is the
         | truest access we have to reality.
        
       | 01GOD wrote:
       | I suspect the op was trolling the hackernews community to
       | intentionally cause a big mathsturbatory circlejerk.
       | 
       | Golf clap on how amazingly effective that was.
        
       | MiddleEndian wrote:
       | https://www.smbc-comics.com/comic/the-largest-number-2
       | 
       | Tangentially related comic
        
         | dwater wrote:
         | I have a degree in math but the first thing this headline made
         | me thing of was still the "24 is the highest number" sketch
         | from Mr. Show:
         | 
         | https://www.youtube.com/watch?v=RkP_OGDCLY0
         | 
         | Theoretical mathematics is often absurd, maybe that's why I
         | like the sketch so much.
        
       ___________________________________________________________________
       (page generated 2021-07-16 23:02 UTC)